102
Georgia Department of Education November 2014 All rights reserved. Assessment Guide Grade 6 Assessment Guide

Georgia Milestones Grade 6 EOG Assessment Guide

  • Upload
    buithuy

  • View
    222

  • Download
    1

Embed Size (px)

Citation preview

Page 1: Georgia Milestones Grade 6 EOG Assessment Guide

Georgia Department of Education November 2014

All rights reserved.

Assessment Guide

Grade 6

Ass

ess

me

nt

Gu

ide

Page 2: Georgia Milestones Grade 6 EOG Assessment Guide

Georgia Milestones Grade 6 EOG Assessment Guide

Georgia Department of Education November 2014 Page 1 of 101

All rights reserved.

THE GEORGIA MILESTONES ASSESSMENT SYSTEM 2

GEORGIA MILESTONES END-OF-GRADE (EOG) ASSESSMENTS 3

ASSESSMENT GUIDE 3

TESTING SCHEDULE 4

DEPTH OF KNOWLEDGE DESCRIPTORS 4

DEPTH OF KNOWLEDGE SKILLS AND QUESTION CUES 6

SCORES 7

ENGLISH LANGUAGE ARTS (ELA) 8

DESCRIPTION OF TEST FORMAT AND ORGANIZATION 8

CONTENT MEASURED 9

GRADE 6 ENGLISH LANGUAGE ARTS (ELA): DOMAIN STRUCTURES AND CONTENT WEIGHTS 10

ITEM TYPES 11

ENGLISH LANGUAGE ARTS (ELA) EXAMPLE ITEMS 11

ENGLISH LANGUAGE ARTS (ELA) ADDITIONAL SAMPLE ITEMS 21

ENGLISH LANGUAGE ARTS (ELA) ADDITIONAL SAMPLE ITEM KEYS 34

ENGLISH LANGUAGE ARTS (ELA) EXAMPLE SCORING RUBRICS AND EXEMPLAR RESPONSES 36

ENGLISH LANGUAGE ARTS (ELA) WRITING RUBRICS 40

MATHEMATICS 46

DESCRIPTION OF TEST FORMAT AND ORGANIZATION 46

CONTENT MEASURED 47

GRADE 6 MATHEMATICS: DOMAIN STRUCTURES AND CONTENT WEIGHTS 48

ITEM TYPES 49

MATHEMATICS EXAMPLE ITEMS 49

MATHEMATICS ADDITIONAL SAMPLE ITEMS 53

MATHEMATICS ADDITIONAL SAMPLE ITEM KEYS 61

MATHEMATICS EXAMPLE SCORING RUBRICS AND EXEMPLAR RESPONSES 63

SCIENCE 67

DESCRIPTION OF TEST FORMAT AND ORGANIZATION 67

CONTENT MEASURED 68

GRADE 6 SCIENCE: DOMAIN STRUCTURES AND CONTENT WEIGHTS 69

ITEM TYPES 70

SCIENCE EXAMPLE ITEMS 70

SCIENCE ADDITIONAL SAMPLE ITEMS 76

SCIENCE ADDITIONAL SAMPLE ITEM KEYS 82

SOCIAL STUDIES 85

DESCRIPTION OF TEST FORMAT AND ORGANIZATION 85

CONTENT MEASURED 86

GRADE 6 SOCIAL STUDIES: DOMAIN STRUCTURES AND CONTENT WEIGHTS 87

ITEM TYPES 88

SOCIAL STUDIES EXAMPLE ITEMS 88

SOCIAL STUDIES ADDITIONAL SAMPLE ITEMS 94

SOCIAL STUDIES ADDITIONAL SAMPLE ITEM KEYS 100

Page 3: Georgia Milestones Grade 6 EOG Assessment Guide

Georgia Milestones Grade 6 EOG Assessment Guide

Georgia Department of Education November 2014 Page 2 of 101

All rights reserved.

THE GEORGIA MILESTONES ASSESSMENT SYSTEM The purpose of the Georgia Student Assessment Program is to measure student achievement of the

state-adopted content standards and inform efforts to improve teaching and learning. Results of the

assessment program are utilized to identify students failing to achieve mastery of content, to provide

educators with feedback about instructional practice, and to assist school districts in identifying

strengths and weaknesses in order to establish priorities in planning educational programs.

The State Board of Education is required by Georgia law (O.C.G.A. §20-2-281) to adopt assessments

designed to measure student achievement relative to the knowledge and skills set forth in the state-

adopted content standards. The Georgia Milestones Assessment System (Georgia Milestones) fulfills this

requirement and, as a key component of Georgia’s Student Assessment Program, is a comprehensive

summative assessment program spanning grade 3 through high school. Georgia Milestones measures

how well students have learned the knowledge and skills outlined in the state-adopted content

standards in Language Arts, Mathematics, Science, and Social Studies. Students in grades 3–8 take an

end-of-grade assessment in each content area, while high school students take an end-of-course

assessment for each of the eight courses designated by the State Board of Education. In accordance with

State Board Rule, Georgia Milestones end-of-course measures serve as the final exams for the specified

high school courses.

The main purpose of Georgia Milestones is to inform efforts to improve student achievement by

assessing student performance on the standards specific to each course or subject/grade tested.

Specifically, Georgia Milestones is designed to provide students and their parents with critical

information about the students’ achievement and, importantly, their preparedness for the next

educational level. The assessment system is a critical informant of the state’s accountability measure,

the College and Career Ready Performance Index (CCRPI), providing an important gauge about the

quality of the educational services and opportunities provided throughout the state. The ultimate goal

of Georgia’s assessment and accountability system is to ensure that all students are provided the

opportunity to engage with high-quality content standards, receive high-quality instruction predicated

upon those standards, and are positioned to meet high academic expectations.

Features of the Georgia Milestones Assessment System include:

open-ended (constructed-response) items in Language Arts and Mathematics (all grades and

courses);

a writing component (in response to passages read by students) at every grade level and course

within the Language Arts assessment;

norm-referenced items in all content areas and courses to complement the criterion-referenced

information and to provide a national comparison; and

a transition to online administration over time, with online administration considered the

primary mode of administration and paper/pencil as a back-up until the transition is complete.

The primary mode of administration for the Georgia Milestones program is online, with the goal of

completing the transition from paper/pencil within five years after the inaugural administration (i.e., the

Page 4: Georgia Milestones Grade 6 EOG Assessment Guide

Georgia Milestones Grade 6 EOG Assessment Guide

Georgia Department of Education November 2014 Page 3 of 101

All rights reserved.

2014–2015 school year). Paper/pencil test materials (such as Braille) will remain available for students

with disabilities who may require them in order to access the assessment.

Georgia Milestones follows guiding principles to help ensure that the assessment system:

is sufficiently challenging to ensure Georgia students are well positioned to compete with other

students across the United States and internationally;

is intentionally designed across grade levels to send a clear signal of student academic progress

and preparedness for the next level, be it the next grade level, course, or college or career;

is accessible to all students, including those with disabilities or limited English proficiency, at all

achievement levels;

supports and informs the state’s educator effectiveness initiatives, ensuring items and forms are

appropriately sensitive to quality instructional practices; and

accelerates the transition to online administration, allowing—over time—for the inclusion of

innovative technology-enhanced items.

Georgia Milestones End-of-Grade (EOG) Assessments

As previously mentioned, Georgia law (§20-2-281) mandates that the State Board of Education adopt

annual measures of student achievement in the content areas of English Language Arts (ELA),

Mathematics, Science, and Social Studies in grades 3 through 8. Students must participate in the Georgia

Milestones content areas measured at the end of each grade in which they are enrolled. State law

further mandates that student achievement in reading, as measured as a component of the Georgia

Milestones English Language Arts (ELA) EOG assessment, be utilized in promotion and retention

decisions for students in grades 3, 5, and 8, while student achievement in mathematics, as measured by

the Georgia Milestones Mathematics EOG assessment, be considered in grades 5 and 8. Students who

fail to demonstrate grade-level achievement on these measures must receive remediation and be

offered an opportunity for a retest prior to consideration for promotion to grades 4, 6, and 9 (§20-2-283

and State Board of Education Rule 160-4-2-.11).

Results of the EOG assessments, according to the legislated and identified purposes, must:

provide a valid measure of student achievement of the state content standards across the full achievement continuum;

provide a clear signal of each student’s preparedness for the next educational level (i.e., grade);

allow for the detection of the academic progress made by each student from one assessed grade to the next;

be suitable for use in promotion and retention decisions at grades 3 (reading), 5 (reading and mathematics), and 8 (reading and mathematics);

support and inform educator effectiveness measures; and

inform state and federal accountability measures at the school, district, and state levels.

Assessment Guide

The Georgia Milestones Grade 6 EOG Assessment Guide is provided to acquaint Georgia educators and

other stakeholders with the structure and content assessed by the tests. Importantly, this guide is not

intended to inform instructional planning. It is essential to note that there are a small number of content

Page 5: Georgia Milestones Grade 6 EOG Assessment Guide

Georgia Milestones Grade 6 EOG Assessment Guide

Georgia Department of Education November 2014 Page 4 of 101

All rights reserved.

standards that are better suited for classroom or individual assessment rather than large-scale

summative assessment. While those standards are not included on the tests, and therefore are not

included in this Assessment Guide, the knowledge, concepts, and skills inherent in those standards are

often required for the mastery of the standards that are assessed. Failure to attend to all content

standards within a content area can limit a student’s opportunity to learn and show what he or she

knows and can do on the assessments.

The Georgia Milestones Grade 6 EOG Assessment Guide is in no way intended to substitute for the

state-mandated content standards; it is provided to help educators better understand the structure and

content of the assessments, but is not all encompassing of the knowledge, concepts and skills covered in

grade 6 or assessed on the tests. The state-adopted content standards and associated standards-based

instructional resources, such as the Content Frameworks, should be used to plan instruction. This

Assessment Guide can serve as a supplement to those resources, in addition to any locally developed

resources, but should not be used in isolation. In principle, this Assessment Guide is intended to be

descriptive of the assessment program and should not be considered all-inclusive. The state-adopted

content standards are located at www.georgiastandards.org.

TESTING SCHEDULE

The Georgia Milestones Grade 6 EOG assessment is offered during the Main Administration each spring

and one Summer Administration for retests. Please note that there will be no retest administrations

during the 2014–2015 school year.

Students will take the Georgia Milestones Grade 6 EOG assessment on days specified by their local

school district during the testing window. Each district determines a local testing window within the

state-designated testing window.

DEPTH OF KNOWLEDGE DESCRIPTORS

Items found on the Georgia Milestones assessments, including the Grade 6 EOG assessment, are

developed with a particular emphasis on cognitive complexity, or Depth of Knowledge (DOK). DOK is

measured on a scale of 1 to 4 and refers to the level of cognitive demand required to complete a task (or

in this case, an assessment item). The higher the level, the more complex the assessment; however,

higher levels do not necessarily mean more difficult items. For instance, a question can have a low DOK

but a medium or even high difficulty level. Conversely, a DOK 4 question may have a low difficulty level

but still require a great deal of cognitive thinking (e.g., analyzing and synthesizing information instead of

just recalling it). The following descriptions and table show the expectations of the four DOK levels in

greater detail.

Level 1 (Recall of Information) generally requires students to identify, list, or define, often asking them

to recall who, what, when, and where. Consequently, this level usually asks students to recall facts,

terms, concepts, and trends and may ask them to identify specific information contained in documents,

excerpts, quotations, maps, charts, tables, graphs, or illustrations. Items that require students to

Page 6: Georgia Milestones Grade 6 EOG Assessment Guide

Georgia Milestones Grade 6 EOG Assessment Guide

Georgia Department of Education November 2014 Page 5 of 101

All rights reserved.

“describe” and/or “explain” could be classified at Level 1 or Level 2 depending on what is to be

described and/or explained. A Level 1 “describe” and/or “explain” would require students to recall,

recite, or reproduce information.

Level 2 (Basic Reasoning) includes the engagement of some mental processing beyond recalling or

reproducing a response. A Level 2 “describe” and/or “explain” would require students to go beyond a

description or explanation of recalled information to describe and/or explain a result or “how” or “why.”

Level 3 (Complex Reasoning) requires reasoning, using evidence, and thinking on a higher and more

abstract level than Level 1 and Level 2. Students will go beyond explaining or describing “how and why”

to justifying the “how and why” through application and evidence. Level 3 questions often involve

making connections across time and place to explain a concept or “big idea.”

Level 4 (Extended Reasoning) requires the complex reasoning of Level 3 with the addition of planning,

investigating, applying significant conceptual understanding, and/or developing that will most likely

require an extended period of time. Students should be required to connect and relate ideas and

concepts within the content area or among content areas in order to be at this highest level. The

distinguishing factor for Level 4 would be evidence through a task, product, or extended response that

the cognitive demands have been met.

The table on the next page identifies skills that students will need to demonstrate at each DOK level,

along with sample question cues appropriate for each level.

Page 7: Georgia Milestones Grade 6 EOG Assessment Guide

Georgia Milestones Grade 6 EOG Assessment Guide

Georgia Department of Education November 2014 Page 6 of 101

All rights reserved.

Depth of Knowledge Skills and Question Cues

Level Skills Demonstrated Question Cues

Recall of Information

Level 1

Make observations

Recall information

Recognize formulas, properties, patterns, processes

Know vocabulary, definitions

Know basic concepts

Perform one-step processes

Translate from one representation to another

Identify relationships

Tell what, when, or where

Find

List

Define

Identify; label; name

Choose; select

Compute; estimate

Express as

Read from data displays

Order

Basic Reasoning

Level 2

Apply learned information to abstract and real-life situations

Use methods, concepts, theories in abstract and real life situations

Perform multi-step processes

Solve problems using required skills or knowledge (requires more than habitual response)

Make a decision about how to proceed

Identify and organize components of a whole

Extend patterns

Identify/describe cause and effect

Recognize unstated assumptions, make inferences

Interpret facts

Compare or contrast simple concepts/ideas

Apply

Calculate; solve

Complete

Describe

Explain how; demonstrate

Construct data displays

Construct; draw

Analyze

Extend

Connect

Classify

Arrange

Compare; contrast

Complex Reasoning

Level 3

Solve an open-ended problem with more than one correct answer

Create a pattern

Generalize from given facts

Relate knowledge from several sources

Draw conclusions

Make predictions

Translate knowledge into new contexts

Compare and discriminate between ideas

Assess value of methods, concepts, theories, processes, formulas

Make choices based on a reasoned argument

Verify the value of evidence, information, numbers, data

Plan; prepare

Predict

Create; design

Ask “what if?” questions

Generalize

Justify; explain why; support; convince

Assess

Rank; grade

Test; judge

Recommend

Select

Conclude

Page 8: Georgia Milestones Grade 6 EOG Assessment Guide

Georgia Milestones Grade 6 EOG Assessment Guide

Georgia Department of Education November 2014 Page 7 of 101

All rights reserved.

Level Skills Demonstrated Question Cues

Extended Reasoning

Level 4

Analyze and synthesize information from multiple sources

Examine and explain alternative perspectives across a variety of sources

Describe and illustrate how common themes are found across texts from different cultures

Apply mathematical models to illuminate a problem or situation

Design a mathematical model to inform and solve a practical or abstract situation

Combine and synthesize ideas into new concepts

Design

Connect

Synthesize

Apply concepts

Critique

Analyze

Create

Prove

SCORES

Students will receive an EOG scale score, an achievement level designation, and a number correct out of

the number possible on items aligned to the state content standards. Students will also receive scores

on norm-referenced items that allow comparison to a national group of students. Additional information

on the items contributing to these scores is found in the Description of Test Format and Organization

section for English Language Arts (ELA), Mathematics, Science, and Social Studies.

Selected-response items are machine scored. The Science and Social Studies assessments consist of only

selected-response items. However, the English Language Arts (ELA) assessment consists of a variety of

item types that contribute to the student’s score, including selected-response, constructed-response,

extended constructed-response, and extended writing-response. Likewise, the Mathematics assessment

consists of selected-response, constructed-response, and extended constructed-response items. Items

that are not machine scored—i.e., constructed-response, extended constructed-response, and extended

writing-response items—require rubrics for manual scoring.

Page 9: Georgia Milestones Grade 6 EOG Assessment Guide

Georgia Milestones Grade 6 EOG Assessment Guide | English Language Arts (ELA)

Georgia Department of Education November 2014 Page 8 of 101

All rights reserved.

ENGLISH LANGUAGE ARTS (ELA)

Description of Test Format and Organization

The Georgia Milestones EOG assessment is primarily a criterion-referenced test, designed to provide

information about how well a student has mastered the grade-level state-adopted content standards in

English Language Arts (ELA). Each student will receive one of four proficiency levels, depending on how

well the student has mastered the content standards. In addition to criterion-referenced information,

the Georgia Milestones measures will also include a limited sample of nationally norm-referenced items

to provide a signal of how Georgia students are achieving relative to their peers nationally. The norm-

referenced information provided is supplementary to the criterion-referenced proficiency designation

and will not be utilized in any manner other than to serve as a barometer of national comparison. Only

the criterion-referenced scores and proficiency designations will be utilized in the accountability metrics

associated with the assessment program (such as student growth measures, educator effectiveness

measures, or the CCRPI).

The Grade 6 English Language Arts (ELA) EOG assessment consists of a total of 60 items, 54 of which are

operational items (and contribute to a student’s criterion-referenced and/or norm-referenced score)

and 6 of which are field test items (newly written items that are being tried out and do not contribute to

the student’s score). The criterion-referenced score, and proficiency designation, is comprised of 44

items, for a total of 55 points. Students will respond to a variety of item types, including selected-

response, constructed-response, extended constructed-response, and extended writing-response items.

Of the 54 operational items, 20 will be norm-referenced and will provide a national comparison in the

form of a national percentile rank. Ten of the items have been verified as aligned to the course content

standards by Georgia educators and will therefore contribute to the criterion-referenced proficiency

designation. The other 10 items will contribute only to the national percentile rank and be provided as

supplemental information. Only items that are aligned to the state-adopted content standards will be

utilized to inform the criterion-referenced score.

With the inclusion of the norm-referenced items, students may encounter items for which they have not

received direct instruction. These items will not contribute to the student’s criterion-referenced

proficiency designation; only items that align to the course content standards will contribute to the

criterion-referenced score. Students should be instructed to try their best should they ask about an item

that is not aligned to the content they have learned as part of the course.

Page 10: Georgia Milestones Grade 6 EOG Assessment Guide

Georgia Milestones Grade 6 EOG Assessment Guide | English Language Arts (ELA)

Georgia Department of Education November 2014 Page 9 of 101

All rights reserved.

Grade 6 English Language Arts (ELA) EOG Assessment Design

Description Number of

Items Points for CR1 Score

Points for NRT2 Feedback

CR Selected-Response Items 30 30 0

NRT Selected-Response Items 203 104 20

CR Constructed-Response Items 3 8 0

CR Extended Writing-Response Items

1 7 0

CR Field Test Items 6 0 0

Total Items/Points5 60 55 20 1CR—Criterion-Referenced: items aligned to state-adopted content standards 2NRT—Norm Referenced Test: items that will yield a national comparison; may or may not be aligned to state-adopted content standards 3Of these items, 10 will contribute to both the CR scores and NRT feedback. The other 10 of these items will contribute to NRT feedback only and will not impact the student’s proficiency designation, scale score, or grade conversion. 4Alignment of national NRT items to course content standards was verified by a committee of Georgia educators. Only approved, aligned NRT items will contribute to a student’s CR proficiency designation, scale score, and grade conversion score. 5Total number of items contributing to CR score: 44; total points: 55; total number of items contributing to NRT feedback: 20; total points: 20

The test will be given in three sections. Students may have up to 70 minutes per section to complete

Sections 1 and 2. Students will be given a maximum of 90 minutes to complete Section 3, which includes

the extended writing-response. The total estimated testing time for the Grade 6 English Language Arts

(ELA) EOG assessment ranges from approximately 190 to 230 minutes. Total testing time describes the

amount of time students have to complete the assessment. It does not take into account the time

required for the test examiner to complete pre-administration and post-administration activities (such

as reading the standardized directions to students). Sections 1 and 2 must be scheduled to be

administered on the same day in one test session following the district’s testing protocols for the EOG

measures (in keeping with state guidance). Section 3, which focuses on writing, must be administered

on a separate day following the completion of Sections 1 and 2.

Content Measured

The Grade 6 English Language Arts (ELA) assessment will measure the standards that are enumerated

for Grade 6 as described on www.georgiastandards.org.

The content of the assessment is organized into two groupings, or domains, of standards for the

purposes of providing feedback on student performance. A content domain is a reporting category that

broadly describes and defines the content of the course, as measured by the EOG assessment. The

standards for Grade 6 English Language Arts (ELA) are grouped into two domains: Reading/Vocabulary

and Writing/Language. Each domain was created by organizing standards that share similar content

characteristics. The content standards describe the level of expertise that Grade 6 English Language Arts

(ELA) educators should strive to develop in their students. Educators should refer to the content

Page 11: Georgia Milestones Grade 6 EOG Assessment Guide

Georgia Milestones Grade 6 EOG Assessment Guide | English Language Arts (ELA)

Georgia Department of Education November 2014 Page 10 of 101

All rights reserved.

standards for a full understanding of the knowledge, concepts, and skills subject to be assessed on the

EOG assessment.

The approximate proportional number of points associated with each domain is shown in the following

table. A range of cognitive levels will be represented on the Grade 6 English Language Arts (ELA) EOG

assessment. Educators should always use the content standards when planning instruction.

Grade 6 English Language Arts (ELA): Domain Structures and Content Weights

Domain Standard Approximate

Weight

Reading and Vocabulary

ELACC6RI1 ELACC6RI2 ELACC6RI3 ELACC6RI4 ELACC6RI5 ELACC6RI6 ELACC6RI7 ELACC6RI8 ELACC6RI9 ELACC6RL1

ELACC6RL2 ELACC6RL3 ELACC6RL4 ELACC6RL5 ELACC6RL6 ELACC6RL9 ELACC6L4

(4a, 4b, 4c) ELACC6L5

(5a, 5b, 5c)

53%

Writing and Language

ELACC6W1 (1a, 1b, 1c, 1d, 1e)

ELACC6W2 (2a, 2b, 2c, 2d, 2e, 2f)

ELACC6W3 (3a, 3b, 3c, 3d, 3e)

ELACC6W4 ELACC6W7

ELACC6W8 ELACC6W9 ELACC6L1

(1a, 1b, 1c, 1d, 1e) ELACC6L2 (2a, 2b)

ELACC6L3 (3a, 3b)

47%

Page 12: Georgia Milestones Grade 6 EOG Assessment Guide

Georgia Milestones Grade 6 EOG Assessment Guide | English Language Arts (ELA)

Georgia Department of Education November 2014 Page 11 of 101

All rights reserved.

Item Types

The English Language Arts (ELA) portion of the Grade 6 EOG assessment consists of selected-response,

constructed-response, extended constructed-response, and extended writing-response items.

A selected-response item, sometimes called a multiple-choice item, is defined as a question, problem, or

statement that appears on a test followed by several answer choices, sometimes called options or

response choices. The incorrect choices, called distractors, usually reflect common errors. The student’s

task is to choose, from the alternatives provided, the best answer to the question posed in the stem (the

question). The English Language Arts (ELA) selected-response items will have four answer choices.

A constructed-response item asks a question and solicits the student to provide a response he or she

constructs on his or her own, as opposed to selecting from options provided. The constructed-response

items on the EOG assessment will be worth two points. Partial credit may be awarded.

An extended constructed-response item is a specific type of constructed-response item that elicits a

longer, more detailed response from the student than a two-point constructed-response item. The

extended constructed-response items on the EOG assessment will be worth four points. For English

Language Arts (ELA), the student will respond to a narrative prompt based on a passage the student has

read, and the response will be scored for the Writing/Language domain. Partial credit may be awarded.

The extended writing-response items require students to produce arguments or develop an informative

response. The extended writing-response, or writing task, includes two passages, three selected-

response items, and one constructed-response item that scaffold students’ understanding of the

passage(s). Two of the selected-response items will address each of the passages separately. One

selected-response item and the constructed-response item will address both of the passages together.

All four items contribute to the Reading/Vocabulary domain. These items will be followed by an

extended writing-prompt, which requires the student to draw from reading experiences when writing an

essay response and to cite evidence from the passage(s) to support claims and conclusions in the essay.

The writing task is worth seven points.

English Language Arts (ELA) Example Items

Example items, which are representative of three DOK levels across various Grade 6 English Language

Arts (ELA) content domains, are provided on the following pages. All example and sample items

contained in this guide are the property of the Georgia Department of Education.

Page 13: Georgia Milestones Grade 6 EOG Assessment Guide

Georgia Milestones Grade 6 EOG Assessment Guide | English Language Arts (ELA)

Georgia Department of Education November 2014 Page 12 of 101

All rights reserved.

Example Items 1 and 2

Read the article “The Hermit Crab” and answer questions 1 and 2.

The Hermit Crab The word hermit is used to describe someone or something that lives alone. In the case of the hermit crab, however, nothing could be further from the truth! Though each hermit crab has its own shell, hermit crabs like to socialize and live in packs. One of the many interesting things about the hermit crab is its body, which is segmented. This means that the hermit crab’s body has different parts, like an insect’s body, rather than one part, like a snake’s body. The front half of the hermit crab’s body is covered in an exoskeleton, or hard outer skin. The lower half of the hermit crab’s body is not covered with an exoskeleton. Thus, it is more fragile than the other parts of the crab’s body. The only protection for this soft part of its body is its shell. However, hermit crabs are not born with shells. They spend a great deal of time searching for abandoned shells that they can squeeze into. The hermit crab keeps its shell until it grows too large for it. Then it leaves to go find another. Though hermit crabs are very particular, they often select objects other than shells to crawl into. For example, they have been known to crawl into small cans. The hermit crab has two front claws that are different sizes and have different purposes. The left claw is large and is used to defend the crab against predators. This claw can also be used to grab objects or to balance when the crab is using its other claw. The right claw is smaller than the left and is used to grasp food. When the hermit crab searches for food, it uses its antennae to smell and taste. The antennae are also used to feel objects. The hermit crab can see in many different directions with its compound eyes. Compound means the eyes have many lenses. The eyes stick out from the rest of the hermit crab’s body because they are at the end of a long body part called an eyestalk.

Page 14: Georgia Milestones Grade 6 EOG Assessment Guide

Georgia Milestones Grade 6 EOG Assessment Guide | English Language Arts (ELA)

Georgia Department of Education November 2014 Page 13 of 101

All rights reserved.

Though the hermit crab is selective about its shell, it is not picky when it comes to the type of food it will eat. The hermit crab tends to eat anything that is lying around. It will eat algae, sea plants, fish, and vegetables. It will also eat decaying matter that has washed ashore. A hermit crab may even eat its own skin once it molts, or sheds, it! Hermit crabs are found in warm, tropical places such as the Caribbean, South America, Central America, and Australia. They are born in water but move to land once they mature. They prefer to live in small, cozy places such as within groups of rocks or under exposed tree roots. Most hermit crab colonies contain around 100 of the little creatures. They tend to pile on top of each other to sleep and to travel in packs. This is why most experts recommend that if you keep a hermit crab as a pet, you should have more than one. (See the box below for more information.) Though hermit crabs are less common as pets, they are a fascinating choice to consider. If you do keep some hermit crabs as pets, I think you will agree that they are wonderful!

Hermit Crabs as Pets

Hermit crabs make great pets. If you would like to purchase and raise hermit crabs as

pets, follow these simple steps:

• Purchase an aquarium that is at least ten gallons.

• Set up a heater underneath the tank, and be sure to keep the temperature

of the aquarium between seventy and eighty-five degrees Fahrenheit.

• Place sand or coconut fiber in the tank so the hermit crabs have a place to

burrow.

• Arrange decorations in the tank, such as plastic plants.

• Put the hermit crabs in their new home.

• Give the hermit crabs access to both fresh food and water.

• Clean the aquarium regularly.

Page 15: Georgia Milestones Grade 6 EOG Assessment Guide

Georgia Milestones Grade 6 EOG Assessment Guide | English Language Arts (ELA)

Georgia Department of Education November 2014 Page 14 of 101

All rights reserved.

Example Item 1

DOK Level: 2

English Language Arts (ELA) Grade 6 Content Domain: Reading and Vocabulary

Standard: ELACC6L4. Determine or clarify the meaning of unknown and multiple-meaning words and

phrases based on grade 6 reading and content, choosing flexibly from a range of strategies.

Which of these is the meaning of mature in the sentence?

A grow useful B develop fully C grow curious D become tired

Correct Answer: B

Explanation of Correct Answer: The correct answer is choice (B) develop fully. This sentence is

explaining the growth cycle of hermit crabs, so "mature" references their full development. Choice (A) is

incorrect because maturing does not mean becoming useful. Choice (C) is incorrect because "mature" is

referring to physical growth. Choice (D) is incorrect because nothing indicates that hermit crabs become

tired when they mature.

They are born in water but move to land once they mature.

Page 16: Georgia Milestones Grade 6 EOG Assessment Guide

Georgia Milestones Grade 6 EOG Assessment Guide | English Language Arts (ELA)

Georgia Department of Education November 2014 Page 15 of 101

All rights reserved.

Example Item 2

DOK Level: 3

English Language Arts (ELA) Grade 6 Content Domain: Reading and Vocabulary

Standard: ELACC6RI5. Analyze how a particular sentence, paragraph, chapter, or section fits into the

overall structure of a text and contributes to the development of the ideas.

How does the last paragraph of the passage affect the passage as a whole? Use details from the

passage to support your answer.

Scoring Rubric

Points Description

2

The response achieves the following:

gives sufficient evidence of the ability to analyze how a paragraph fits into the overall structure of a text and to explain how it contributes to the development of ideas

includes specific examples/details that make clear reference to the text

adequately explains how a paragraph fits into the overall structure of a text and how it contributes to the development of ideas with clearly relevant information based on the text

1

The response achieves the following:

gives limited evidence of the ability to analyze how a paragraph fits into the overall structure of a text or to explain how it contributes to the development of ideas

includes vague/limited examples/details that make reference to the text

explains how a paragraph fits into the overall structure of a text and how it contributes to the development of ideas with vague/limited information based on the text

0

The response achieves the following:

gives no evidence of the ability to analyze how a paragraph fits into the overall structure of a text or to explain how it contributes to the development of ideas

OR

analyzes how a paragraph fits into the overall structure of a text, but includes no examples or no examples/details that make reference to the text

OR

analyzes how a paragraph fits into the overall structure of a text, but includes no explanation of its contributions or no relevant information from the text

Page 17: Georgia Milestones Grade 6 EOG Assessment Guide

Georgia Milestones Grade 6 EOG Assessment Guide | English Language Arts (ELA)

Georgia Department of Education November 2014 Page 16 of 101

All rights reserved.

Example Item 2

Exemplar Response

Points Awarded

Response

2

The last paragraph brings the passage full circle by supporting the information in the first paragraph. It explains how hermit crab colonies have 100 crabs, and if you keep them as pets, "you should have more than one." This supports the claim in the first paragraph that hermit crabs "like to socialize and live in packs."

1 The last paragraph supports the information in the first paragraph by explaining how hermit crabs would rather "socialize and live in packs" than be alone.

0 The last paragraph talks about the same things as what the first paragraph talks about.

Page 18: Georgia Milestones Grade 6 EOG Assessment Guide

Georgia Milestones Grade 6 EOG Assessment Guide | English Language Arts (ELA)

Georgia Department of Education November 2014 Page 17 of 101

All rights reserved.

Example Item 3

DOK Level: 4

English Language Arts (ELA) Grade 6 Content Domain: Writing and Language

Standard: ELACC6W1. Use words, phrases, and clauses to clarify the relationships among claim(s) and

reasons.

In this section, you will read about an ongoing public debate as to whether television and movies

reflect society or influence it. You will write an argumentative essay supporting either side of the

debate in which you argue that television and movies reflect society or that they influence it.

Before you begin planning and writing, read the two texts:

1. “Movies and Television: A Reflection”

2. “Positive Influence, Please!”

As you read the texts, think about what details from the texts you might use in your argumentative

essay.

Page 19: Georgia Milestones Grade 6 EOG Assessment Guide

Georgia Milestones Grade 6 EOG Assessment Guide | English Language Arts (ELA)

Georgia Department of Education November 2014 Page 18 of 101

All rights reserved.

Movies and Television: A Reflection

Movies and television reflect society because they tell stories about our social world. People enjoy stories about that world because they connect with stories about themselves. They also enjoy stories about other people, especially if it involves cheering for people who work their way out of tough situations. Consider a television situation comedy about a middle-class family. The parents and their three children have many bad days that could be drawn from many true-life experiences. Their exaggerated ups and downs are humorous, but together the family survives. The show reflects society by showing a world to which most people can relate. Sometimes the characters in the story are not people. Consider a cartoon movie about a lion family. The plot seems to be based on real animals. Male lions are competing to be the head of a family group. However, these lions also have names. They talk to each other. They sing. They show human emotions. In other words, the plot is really a human drama. It reflects society by portraying true family relationships. Sometimes writers include elements of modern society in movies and television shows that are set in the past. They believe this helps more viewers imagine being part of the story. For example, consider a movie about a pirate who is both foreign and familiar. He can sail a tall ship, but he likes to stretch rules. He appears to be from the 1700s, but he behaves much more like a modern teen. Many parts of his character are clearly drawn from the experiences of real people. The movie reflects both historical and modern society. These three examples are similar to movies that have been very popular with their audiences. They show that the best movie and television writers draw from the experiences of real people to tell their stories. When the characters seem real, the plots imitate life, and the stories reflect our society.

Positive Influence, Please!

Movies and television have the power to change society. Why? Because people like to imitate each other. In fact, people are wired to imitate. Babies imitate the looks on their parents’ faces. Young children learn how to talk by imitation. Even teens and adults learn by imitation. They pick up social cues. They copy ways of speaking. They copy each other’s clothes. They listen to each other’s songs. They watch the latest movies and shows. Partly they do this to explore the world in a safe way. Partly they do it to fit in with their peers. Mostly they do it by instinct, without thinking. Millions of people watch movies and shows. Because the characters on the screen are also people, they can prompt the audience to imitate them. They can change people’s behavior. For example, in the 1970s, women all over the world copied the hairdo of an actress in television series. Anyone whose hair could hold the famous flip wore the style. Similarly, in the 1990s many young children learned the moves of a group of superheroes who appeared in both a television series and several full-length movies.

Page 20: Georgia Milestones Grade 6 EOG Assessment Guide

Georgia Milestones Grade 6 EOG Assessment Guide | English Language Arts (ELA)

Georgia Department of Education November 2014 Page 19 of 101

All rights reserved.

Just like children, teens and adults copy speech patterns from movies and television shows. This imitation of language appears to have a more lasting effect. For example, because it is shocking and adds drama, characters on some shows are disrespectful. People who mistake movie and television scripts for real life copy these characters. As a result, disrespectful language is creeping into everyday speech. The same characters behave badly toward each other. They have changed some people’s ideas about how to behave in family groups. However, all is not lost! There are situation comedies and movies that have changed society for the better. Several popular high school dramas show people from different social groups getting along and making friends. These same shows give teens good ideas about how to handle social problems and relationships.

Now that you have read “Movies and Television: A Reflection” and “Positive Influence, Please!”,

create a plan for your argumentative essay.

Think about ideas, facts, definitions, details, and other information and examples you want to use.

Think about how you will introduce your topic and what the main topic will be for each paragraph. Be

sure to identify the sources by title or number when using details or facts directly from the sources.

Write an argumentative essay supporting either side of the debate in which you argue that television

and movies reflect society or that they influence it.

Now write your argumentative essay. Be sure to:

Introduce your claim.

Support your claim with logical reasoning and relevant evidence from the texts.

Acknowledge and address alternate or opposing claims.

Organize the reasons and evidence logically.

Use words, phrases, and clauses to connect your ideas and to clarify the relationships among

claims, counterclaims, reasons, and evidence.

Establish and maintain a formal style.

Provide a concluding statement or section that follows from and supports the argument

presented.

Check your work for correct usage, grammar, spelling, and capitalization.

Page 21: Georgia Milestones Grade 6 EOG Assessment Guide

Georgia Milestones Grade 6 EOG Assessment Guide | English Language Arts (ELA)

Georgia Department of Education November 2014 Page 20 of 101

All rights reserved.

To view the seven point two-trait rubric for a text-based argumentative response, see page 44.

Example of a 7-Point Response:

People enjoy telling stories and hearing stories about people and places in our world. Since movies and

television shows tell stories as a form of entertainment or education, it is difficult to determine if they

influence us or just reflect how we are in our society. The author of the first passage, “Movies and

Television: A Reflection,” makes the strongest case that these stories mirror the way people are. These

stories are told in different ways, using real people or animation, or using animals as characters.

However, the feelings and reactions described reflect feelings we all have as human beings. We may be

influenced by a dramatic plot, but that will not change who we are and how we live our lives.

The second passage, “Positive Influence, Please!”, implies that people are heavily influenced by what

they hear and see in movies and on television shows. It suggests that people change their language and

behavior after viewing dramatic shows. The author is saying that people, by the way they learn, imitate

all they hear and see. That assumption does not give individuals credit for having their own views and

values. People may rethink their worldviews based on new information, but they make a conscious

decision to change.

In all cultures, sharing stories with others is a way these societies maintain their heritage and sense of

self. Therefore, it seems more reasonable to say that movies and television reflect society than to say

that they influence it.

Page 22: Georgia Milestones Grade 6 EOG Assessment Guide

Georgia Milestones Grade 6 EOG Assessment Guide | English Language Arts (ELA)

Georgia Department of Education November 2014 Page 21 of 101

All rights reserved.

English Language Arts (ELA) Additional Sample Items

This section has two parts. The first part is a set of 10 sample items for the English Language Arts (ELA)

portion of the EOG assessment. The second part contains a table that shows for each item the standard

assessed, the DOK level, the correct answer (key), and a rationale/explanation about the key and

distractors. The sample items can be utilized as a mini-test to familiarize students with the item formats

found on the assessment. All example and sample items contained in this guide are the property of the

Georgia Department of Education.

Page 23: Georgia Milestones Grade 6 EOG Assessment Guide

Georgia Milestones Grade 6 EOG Assessment Guide | English Language Arts (ELA)

Georgia Department of Education November 2014 Page 22 of 101

All rights reserved.

Items 1 through 8

Read the poem “The Carpenter” and the story “The Carpenter’s Apprentice” and answer questions 1

through 8.

The Carpenter A house is sketched on paper, Then drawn on plans of blue. But it is the carpenter’s careful labor That makes the dream come true. Calloused hands unroll the blueprint. Keen eyes review with ease. A young man might see a house, But a home the carpenter sees. He cuts and shapes with vision. His goal is understood: He converts the lines and numbers Into lengths of measured wood. Like an artist he wields his hammer, Pounding rhythms to his own beat. He sculpts and forms a framework That painted walls will soon complete. Like an athlete he climbs and balances. Lifting, fastening bulky beams, Building the backbone of a sturdy house, He frames a family’s dreams. Walls and windows, floors and doors, The carpenter adds with care. He knows a fireplace heats a house, But a happy family warms the air. When the home of dreams is ready, An inner smile he then sets free. A young man might see a house, But a home the carpenter sees.

The Carpenter’s Apprentice Before Ben started to work with me, I had advised him to buy a quality set of tools. “Good tools are expensive,” I had told him, “but they will last for years.” On Ben’s first day of work, the head of the cheap hammer he had bought flew off and put a hole in a wall.

Page 24: Georgia Milestones Grade 6 EOG Assessment Guide

Georgia Milestones Grade 6 EOG Assessment Guide | English Language Arts (ELA)

Georgia Department of Education November 2014 Page 23 of 101

All rights reserved.

One Monday as we ate our lunch in the truck outside the Pine Street house, Ben told me that he had made a few phone calls for us. “The lumber yard across town can deliver what we need tomorrow. We could finish the job by noon Friday!” I turned and met his eyes. “I think I told you that I don’t do business with that lumber yard anymore. The last order I got from them had warps, splits, and four-inch knots on every piece.” “But if we wait until the other delivery on Thursday, we won’t finish the job until Monday or Tuesday of next week.” I continued to look at him. Ben was thinking of his plans for the weekend, but I was thinking of our obligation to the house’s owner to do the job right using only quality materials. “Ben,” I sighed, “how would you like to live in this house?” He looked up at me quizzically, as though I was making him an offer. “Would you like to live in this house if it were built your way, using inferior lumber? Would you want to walk around on a floor with warped supports under it and sleep under a roof built with split and knotted wood? We need to do this right, Ben. You can’t build a house twice.” A philosopher, I’m not sure who, once said something to the effect that when you finish building your house, you realize all that you have learned in the process—and you realize, too, that all you have learned you should have known before you started.

Item 1

Which detail from the story suggests that Ben needs to take his apprenticeship more seriously? A Ben is thinking about his plans for the weekend.

B Ben puts a hole in a wall when using his hammer.

C Ben contacts a lumber yard about delivering supplies.

D Ben eats his lunch in a truck away from other workers.

Item 2

In the story, what does the carpenter believe is the MOST important lesson for Ben to learn?

A build a house only once

B use the best tools and materials

C do the work with care and consideration

D know everything before starting a project

Page 25: Georgia Milestones Grade 6 EOG Assessment Guide

Georgia Milestones Grade 6 EOG Assessment Guide | English Language Arts (ELA)

Georgia Department of Education November 2014 Page 24 of 101

All rights reserved.

Item 3

Which theme is shared by the story and the poem?

A Beginning carpenters require training.

B It is good to take pride in your work.

C It is good to ask many questions.

D Carpenters need special tools.

Item 4

What is the MOST LIKELY reason the poet included the second stanza in the poem? Use details from

the poem to support your answer.

Calloused hands unroll the blueprint.

Keen eyes review with ease.

A young man might see a house,

But a home the carpenter sees.

Page 26: Georgia Milestones Grade 6 EOG Assessment Guide

Georgia Milestones Grade 6 EOG Assessment Guide | English Language Arts (ELA)

Georgia Department of Education November 2014 Page 25 of 101

All rights reserved.

Item 5

Use the information in the box below to answer the question.

Which word would BEST replace obligation in the story?

A burden

B charge

C debt

D duty

Item 6

Which word would BEST replace wields in this stanza from the poem?

A holds

B maintains

C shakes

D uses

obligation n. burden, charge, debt, duty

I continued to look at him. Ben was thinking of his plans for the

weekend, but I was thinking of our obligation to the house’s owner

to do the job right using only quality materials.

Like an artist he wields his hammer,

Pounding rhythms to his own beat.

He sculpts and forms a framework

That painted walls will soon complete.

Page 27: Georgia Milestones Grade 6 EOG Assessment Guide

Georgia Milestones Grade 6 EOG Assessment Guide | English Language Arts (ELA)

Georgia Department of Education November 2014 Page 26 of 101

All rights reserved.

Item 7

What does the poet compare a carpenter to in this stanza from the poem?

A a creative person

B a sturdy house

C a reliable tool

D a young man

Like an artist he wields his hammer,

Pounding rhythms to his own beat.

He sculpts and forms a framework

That painted walls will soon complete.

Page 28: Georgia Milestones Grade 6 EOG Assessment Guide

Georgia Milestones Grade 6 EOG Assessment Guide | English Language Arts (ELA)

Georgia Department of Education November 2014 Page 27 of 101

All rights reserved.

Item 8

From the narrator's point of view, finish the story of "The Carpenter's Apprentice." Start with what

Ben might say in response to the narrator. Use details from the poem "The Carpenter" in your answer.

Be sure to use descriptive words and phrases in your writing.

Page 29: Georgia Milestones Grade 6 EOG Assessment Guide

Georgia Milestones Grade 6 EOG Assessment Guide | English Language Arts (ELA)

Georgia Department of Education November 2014 Page 28 of 101

All rights reserved.

Items 9 and 10

In this section, you will read about an ongoing public debate as to whether television and movies

reflect society or influence it. You will write an argumentative essay supporting either side of the

debate in which you argue that television and movies reflect society or that they influence it.

Before you begin planning and writing, read the two texts:

1. “Movies and Television: A Reflection”

2. “Positive Influence, Please!”

As you read the texts, think about what details from the texts you might use in your argumentative

essay.

Movies and Television: A Reflection Movies and television reflect society because they tell stories about our social world. People enjoy stories about that world because they connect with stories about themselves. They also enjoy stories about other people, especially if it involves cheering for people who work their way out of tough situations. Consider a television situation comedy about a middle-class family. The parents and their three children have many bad days that could be drawn from many true-life experiences. Their exaggerated ups and downs are humorous, but together the family survives. The show reflects society by showing a world to which most people can relate. Sometimes the characters in the story are not people. Consider a cartoon movie about a lion family. The plot seems to be based on real animals. Male lions are competing to be the head of a family group. However, these lions also have names. They talk to each other. They sing. They show human emotions. In other words, the plot is really a human drama. It reflects society by portraying true family relationships. Sometimes writers include elements of modern society in movies and television shows that are set in the past. They believe this helps more viewers imagine being part of the story. For example, consider a movie about a pirate who is both foreign and familiar. He can sail a tall ship, but he likes to stretch rules. He appears to be from the 1700s, but he behaves much more like a modern teen. Many parts of his character are clearly drawn from the experiences of real people. The movie reflects both historical and modern society. These three examples are similar to movies that have been very popular with their audiences. They show that the best movie and television writers draw from the experiences of real people to tell their stories. When the characters seem real, the plots imitate life, and the stories reflect our society.

Positive Influence, Please! Movies and television have the power to change society. Why? Because people like to imitate each other. In fact, people are wired to imitate. Babies imitate the looks on their parents’ faces. Young children learn how to talk by imitation.

Page 30: Georgia Milestones Grade 6 EOG Assessment Guide

Georgia Milestones Grade 6 EOG Assessment Guide | English Language Arts (ELA)

Georgia Department of Education November 2014 Page 29 of 101

All rights reserved.

Even teens and adults learn by imitation. They pick up social cues. They copy ways of speaking. They copy each other’s clothes. They listen to each other’s songs. They watch the latest movies and shows. Partly they do this to explore the world in a safe way. Partly they do it to fit in with their peers. Mostly they do it by instinct, without thinking. Millions of people watch movies and shows. Because the characters on the screen are also people, they can prompt the audience to imitate them. They can change people’s behavior. For example, in the 1970s, women all over the world copied the hairdo of an actress in television series. Anyone whose hair could hold the famous flip wore the style. Similarly, in the 1990s many young children learned the moves of a group of superheroes who appeared in both a television series and several full-length movies. Just like children, teens and adults copy speech patterns from movies and television shows. This imitation of language appears to have a more lasting effect. For example, because it is shocking and adds drama, characters on some shows are disrespectful. People who mistake movie and television scripts for real life copy these characters. As a result, disrespectful language is creeping into everyday speech. The same characters behave badly toward each other. They have changed some people’s ideas about how to behave in family groups. However, all is not lost! There are situation comedies and movies that have changed society for the better. Several popular high school dramas show people from different social groups getting along and making friends. These same shows give teens good ideas about how to handle social problems and relationships.

Page 31: Georgia Milestones Grade 6 EOG Assessment Guide

Georgia Milestones Grade 6 EOG Assessment Guide | English Language Arts (ELA)

Georgia Department of Education November 2014 Page 30 of 101

All rights reserved.

Item 9

Which author MOST successfully develops the topic according to his/her purpose using reasoning and

evidence? Use details from BOTH passages to support your answer.

Page 32: Georgia Milestones Grade 6 EOG Assessment Guide

Georgia Milestones Grade 6 EOG Assessment Guide | English Language Arts (ELA)

Georgia Department of Education November 2014 Page 31 of 101

All rights reserved.

Item 10

Now that you have read “Movies and Television: A Reflection” and “Positive Influence, Please!” and

answered a question about what you have read, create a plan for your argumentative essay.

Think about ideas, facts, definitions, details, and other information and examples you want to use.

Think about how you will introduce your topic and what the main topic will be for each paragraph. Be

sure to identify the sources by title or number when using details or facts directly from the sources.

Write an argumentative essay supporting either side of the debate in which you argue that television

and movies reflect society or that they influence it.

Now write your argumentative essay. Be sure to:

Introduce your claim.

Support your claim with logical reasoning and relevant evidence from the texts.

Acknowledge and address alternate or opposing claims.

Organize the reasons and evidence logically.

Use words, phrases, and clauses to connect your ideas and to clarify the relationships among

claims, counterclaims, reasons, and evidence.

Establish and maintain a formal style.

Provide a concluding statement or section that follows from and supports the argument

presented.

Check your work for correct usage, grammar, spelling, and capitalization.

Page 33: Georgia Milestones Grade 6 EOG Assessment Guide

Georgia Milestones Grade 6 EOG Assessment Guide | English Language Arts (ELA)

Georgia Department of Education November 2014 Page 32 of 101

All rights reserved.

Page 34: Georgia Milestones Grade 6 EOG Assessment Guide

Georgia Milestones Grade 6 EOG Assessment Guide | English Language Arts (ELA)

Georgia Department of Education November 2014 Page 33 of 101

All rights reserved.

Page 35: Georgia Milestones Grade 6 EOG Assessment Guide

Georgia Milestones Grade 6 EOG Assessment Guide | English Language Arts (ELA)

Georgia Department of Education November 2014 Page 34 of 101

All rights reserved.

English Language Arts (ELA) Additional Sample Item Keys

Item Standard/ Element

DOK Level Correct Answer

Explanation

1 ELACC6RL1 3 A

The correct answer is choice (A) Ben is thinking about his plans for the weekend. Ben wants to rush his work so he can relax, but he is neglecting the responsibilities of his apprenticeship. Choice (B) is incorrect because making a mistake doesn't imply that Ben doesn't take his apprenticeship seriously. Choice (C) is incorrect because contacting another company can be a sign of taking work seriously. Choice (D) is incorrect because eating lunch away from construction work is likely safer and doesn't show lack of seriousness about work.

2 ELACC6RL1 3 C

The correct answer is choice (C) do the work with care and consideration. Throughout the story, the carpenter constantly tries to teach Ben to value quality, such as explaining why the materials from the lumber yard are unacceptable in his opinion. Choice (A) is incorrect because it is a misunderstanding of the phrase the carpenter uses to try to teach Ben about quality. Choice (B) is incorrect because using the right tools is only part of the carpenter's larger lesson about quality. Choice (D) is incorrect because nothing suggests that the carpenter expects his apprentice to know everything.

3 ELACC6RL2 3 B

The correct answer is choice (B) It is good to take pride in your work. The poem celebrates the work of a carpenter and the story clearly shows that the carpenter values quality. Choice (A) is incorrect because the poem does not mention an apprentice. Choice (C) is incorrect because neither the poem nor the story suggests anything about asking many questions. Choice (D) is incorrect because this is only a minor detail of the story and not a theme.

4 ELACC6RL5 3 N/A See scoring rubric and exemplar responses on page 36.

Page 36: Georgia Milestones Grade 6 EOG Assessment Guide

Georgia Milestones Grade 6 EOG Assessment Guide | English Language Arts (ELA)

Georgia Department of Education November 2014 Page 35 of 101

All rights reserved.

Item Standard/ Element

DOK Level Correct Answer

Explanation

5 ELACC6L4c 2 D

The correct answer is choice (D) duty. Obligations, or duties, are responsibilities that one has to fulfill. Choice (A) is incorrect because obligations are not necessarily burdens. Choice (B) is incorrect because a charge does not refer to responsibilities or duties. Choice (C) is incorrect because nothing in the context of the story mentions money.

6 ELACC6L4a 2 D

The correct answer is choice (D) uses. Wielding something refers to holding it and using it to complete a task. Choice (A) is incorrect because wielding refers to more than just holding. Choice (B) is incorrect because nothing in the context suggests that the carpenter is maintaining something. Choice (C) is incorrect because nothing suggests that the carpenter is shaking the hammer.

7 ELACC6L5a 2 A

The correct answer is choice (A) a creative person. Artists and sculptors are creative people, and the poet is using them to describe the carpenter. Choice (B) is incorrect because nothing in the stanza suggests that the carpenter is a sturdy house. Choice (C) is incorrect because nothing suggests that the carpenter is a sturdy tool. Choice (D) is incorrect because nothing suggests that the carpenter is a young man.

8 ELACC6W3d 4 N/A See exemplar responses on page 37 and the four point holistic rubric on page 41.

9 ELACC6RI8 3 N/A See scoring rubric and exemplar responses on page 38.

10 ELACC6W1c 4 N/A See exemplar response on page 39 and the seven point two-trait rubric beginning on page 44.

Page 37: Georgia Milestones Grade 6 EOG Assessment Guide

Georgia Milestones Grade 6 EOG Assessment Guide | English Language Arts (ELA)

Georgia Department of Education November 2014 Page 36 of 101

All rights reserved.

English Language Arts (ELA) Example Scoring Rubrics and Exemplar Responses

Item 4

Scoring Rubric

Points Description

2

The response achieves the following:

gives sufficient evidence of the ability to analyze how a stanza of a poem fits into the overall structure and to explain how it contributes to the text

includes specific examples/details that make clear reference to the text

adequately explains the purpose and contributions of the stanza of a poem with clearly relevant information based on the text

1

The response achieves the following:

gives limited evidence of the ability to analyze how a stanza of a poem fits into the overall structure and to explain how it contributes to the text

includes vague/limited examples/details that make reference to the text

explains the purpose and contributions of the stanza of a poem with vague/limited information based on the text

0

The response achieves the following:

gives no evidence of the ability to analyze how a stanza of a poem fits into the overall structure or to explain how it contributes to the text

OR

gives an explanation of how a stanza of a poem fits into the overall structure, but includes no examples or no examples/details that make reference to the text

OR

explains the contributions of a stanza of a poem, but includes no relevant information from the text

Exemplar Response

Points Awarded

Response

2

The second stanza shows the carpenter's experience and vision for the project. Because he reviews the blueprint "with ease," it's clear he understands it well. The line "But a home the carpenter sees" shows how he sees something more than just a structure in what he is building. By including this information, the poet shows there is more to the carpenter than someone who is just a builder; a carpenter is a creator.

1 The second stanza shows the carpenter's experience and vision for the project. By including this information, the poet shows there is more to the carpenter than someone who is just a builder; a carpenter is a creator.

0 The second stanza shows the carpenter's experience and vision for the project.

Page 38: Georgia Milestones Grade 6 EOG Assessment Guide

Georgia Milestones Grade 6 EOG Assessment Guide | English Language Arts (ELA)

Georgia Department of Education November 2014 Page 37 of 101

All rights reserved.

Item 8

To view the four point holistic rubric for a text-based narrative response, see page 41.

Exemplar Response

Points Awarded

Response

4

Ben thought about what I had said and then admitted that building a house was more than just putting a bunch of wood together. He said, "You're right. I would want to live in a home, one that is built with skill and vision. One that is a carpenter's dream come true. There's more to this job than just nails and wood."

I nodded in agreement. "There really is. We convert lines and numbers from the blueprint, but we do more than that."

Ben went on: "We are like artists, really. We are sculpting a house, not just building it. We add walls, windows, floors, and doors, and we do it with care. I want to make a house that makes people happy. I promise to do better from now on."

I knew that Ben meant it. There was a gleam in his eyes that showed he believed what he said. "Shall we get to it, then?" I asked.

"Absolutely," Ben replied. He picked up his hammer as if it were the paintbrush of an artist or the chisel of a sculptor.

3

Ben thought about what I had said. Then he admitted that building a house was more than just putting wood together. He said, "You're right. I would want to live in a home, one that is built with skill and is a carpenter's dream. There's more to this job than just nails and wood."

I nodded. "There really is. We convert lines and numbers from the blueprint, but we do more than that."

Ben went on: "We are like artists sculpting a house. We don't just build it. I want to make a house that makes people happy."

I knew that Ben meant it. "Shall we get to it, then?" I asked.

"Absolutely," Ben replied.

2

Ben thought about what I had said. Then he said that building a house was more than putting wood together. He said he would want to live in a home that's a dream.

I said, "We take lines and numbers and make it into a home."

Ben said, "I want to make a house that makes people happy."

1

Ben said that building a house was more than putting wood together. He said he would want to live in a home that's a dream.

I said we make houses into homes.

Ben said he wants to make a house that makes people happy.

0 Ben said he wanted to be an artist with wood.

Page 39: Georgia Milestones Grade 6 EOG Assessment Guide

Georgia Milestones Grade 6 EOG Assessment Guide | English Language Arts (ELA)

Georgia Department of Education November 2014 Page 38 of 101

All rights reserved.

Item 9

Scoring Rubric

Points Description

2

The response achieves the following:

gives sufficient evidence of the ability to evaluate the arguments and claims in two texts and to assess the relevance and soundness of the argument/evidence

includes specific examples/details that make clear reference to the texts

adequately explains the arguments and claims in two texts and assessment of evidence with clearly relevant information based on the texts

1

The response achieves the following:

gives limited evidence of the ability to evaluate the arguments and claims in two texts and to assess the relevance and soundness of the argument/evidence

includes vague/limited examples/details that make reference to the texts

explains the arguments and claims in two texts and assessment of evidence with clearly relevant information based on the texts

0

The response achieves the following:

gives no evidence of the ability to evaluate the arguments and claims in two texts or to assess the relevance and soundness of the argument/evidence

OR

gives arguments and claims in two texts, but includes no examples or no examples/details that make reference to the texts

OR

gives arguments and claims in two texts and/or assessment of evidence, but includes no explanation or no relevant information from the texts

Exemplar Response

Points Awarded

Response

2

In “Movies and Television: A Reflection,” the author gives three concrete examples to support his claim that movies and TV reflect society. He explains how situation comedies draw from real-life experiences that most people watching can relate to. The author even explains that animated cartoons about animals reflect society because the animals display human emotions and interact like humans. Similar to that, the author also explains that modern movies set in the past will even have characters that are more similar to people in modern times. The author says that any story with realistic characters will reflect society.

1

I think the author of the first passage did a better job because her point of view was much clearer and easier to understand. That author stayed focused on the idea that movies and TV shows reflect society. She gave many examples why. The author of the second article seemed to not be sure whether imitating things in movies or on TV is a good idea or not because he explains that some imitation can lead to disrespectful behavior and some can lead to good behavior.

0 I think that movies and TV shows do reflect society.

Page 40: Georgia Milestones Grade 6 EOG Assessment Guide

Georgia Milestones Grade 6 EOG Assessment Guide | English Language Arts (ELA)

Georgia Department of Education November 2014 Page 39 of 101

All rights reserved.

Item 10

To view the seven point two-trait rubric for a text-based argumentative response, see page 44.

Example of a 7-Point Response:

People enjoy telling stories and hearing stories about people and places in our world. Since movies and

television shows tell stories as a form of entertainment or education, it is difficult to determine if they

influence us or just reflect how we are in our society. The author of the first passage, “Movies and

Television: A Reflection,” makes the strongest case that these stories mirror the way people are. These

stories are told in different ways, using real people or animation, or using animals as characters.

However, the feelings and reactions described reflect feelings we all have as human beings. We may be

influenced by a dramatic plot, but that will not change who we are and how we live our lives.

The second passage, “Positive Influence, Please!”, implies that people are heavily influenced by what

they hear and see in movies and on television shows. It suggests that people change their language and

behavior after viewing dramatic shows. The author is saying that people, by the way they learn, imitate

all they hear and see. That assumption does not give individuals credit for having their own views and

values. People may rethink their worldviews based on new information, but they make a conscious

decision to change.

In all cultures, sharing stories with others is a way these societies maintain their heritage and sense of

self. Therefore, it seems more reasonable to say that movies and television reflect society than to say

that they influence it.

Page 41: Georgia Milestones Grade 6 EOG Assessment Guide

Georgia Milestones Grade 6 EOG Assessment Guide | English Language Arts (ELA)

Georgia Department of Education November 2014 Page 40 of 101

All rights reserved.

English Language Arts (ELA) Writing Rubrics

English Language Arts (ELA) items that are not machine scored—i.e., constructed-response, extended

constructed-response, and extended writing-response items—are manually scored using either a holistic

rubric or a two-trait rubric.

Four Point Holistic Rubric

Genre: Narrative

A holistic rubric essentially has one main criterion. On the Georgia Milestones EOG assessment, a holistic

rubric contains a single point scale ranging from zero to four. Each point value represents a qualitative

description of the student’s work. To score an item on a holistic rubric, the scorer or reader need only

choose the description and associated point value that best represents the student’s work. Increasing

point values represent a greater understanding of the content and, thus, a higher score.

Seven Point Two-Trait Rubric

Genre: Informational/Explanatory or Argumentative

A two-trait rubric, on the other hand, is an analytic rubric with two criteria, or traits. On the Georgia

Milestones EOG assessment, a two-trait rubric contains two point scales for each trait ranging from zero

to four on one scale and zero to three on the other. A score is given for each of the two criteria/traits,

for a total of seven possible points for the item. To score an item on a two-trait rubric, a scorer or reader

must choose the description and associated point value for each criteria/trait that best represents the

student’s work. The two scores are added together. Increasing point values represent a greater

understanding of the content and, thus, a higher score.

On the following pages are the rubrics that will be used to evaluate writing on the Georgia Milestones

Grade 6 English Language Arts (ELA) EOG assessment.

Page 42: Georgia Milestones Grade 6 EOG Assessment Guide

Georgia Milestones Grade 6 EOG Assessment Guide | English Language Arts (ELA)

Georgia Department of Education November 2014 Page 41 of 101

All rights reserved.

Four Point Holistic Rubric Genre: Narrative

Description Points Criteria

The Narrative writing task examines the writer’s ability to effectively develop real or imagined experiences or events using effective techniques, descriptive details, and clear event sequences based on a text that has been read.

4 The student’s response is a well-developed narrative that fully develops a real or imagined experience based on a text as a stimulus.

Effectively establishes a situation and introduces a narrator and/or characters

Organizes an event sequence that unfolds naturally

Effectively uses narrative techniques, such as dialogue, description, and pacing to develop rich, interesting experiences, events and/or characters

Uses a variety of words and phrases consistently to convey the sequence of events and signal shifts from one time frame or setting to another

Uses precise words, phrases, and sensory language consistently to convey experiences and events

Provides a conclusion that follows from the narrated experiences or events

Integrates ideas and details from source material effectively

Has very few or no errors in usage and/or conventions that interfere with meaning*

3 The student’s response is a complete narrative that develops a real or imagined experience based on a text as a stimulus.

Establishes a situation and introduces one or more characters

Organizes events in a clear, logical order

Uses narrative techniques, such as dialogue, description, and pacing to develop experiences, events and/or characters

Uses words and/or phrases to indicate sequence of events and signal shifts from one time frame or setting to another

Uses words, phrases, and details to convey experiences and events

Provides an appropriate conclusion

Integrates some ideas and/or details from source material

Has few or no errors in usage and/or conventions that interfere with meaning*

2 The student’s response is an incomplete or oversimplified narrative based on a text as a stimulus. Introduces a vague situation and at least one character.

Organizes events in a sequence but with some gaps or ambiguity

Attempts to use a narrative technique, such as dialogue, description, and pacing to develop experiences, events and/or characters

Uses occasional signal words inconsistently to indicate sequence of events and signal shifts from one time frame or setting to another

Uses some words or phrases inconsistently to convey experiences and events

Provides a weak or ambiguous conclusion

Attempts to integrate ideas or details from source material

Has frequent errors in usage and conventions that sometimes interfere with meaning*

1 The student’s response provides evidence of an attempt to write a narrative based on a text as a stimulus.

Provides a weak or minimal introduction of a situation or a character

May be too brief to demonstrate a complete sequence of events

Shows little or no attempt to use dialogue, description, and pacing to develop experiences, events and/or characters

Uses words that are inappropriate, overly simple, or unclear

Provides few if any words that convey events of events and signal shifts from one time frame or setting to another

Provides a minimal or no conclusion

May use few if any ideas or details from source material

Has frequent major errors in usage and conventions that interfere with meaning*

0 The response is completely irrelevant or incorrect, or there is no response.

The student merely copies the text in the prompt.

The student copies so much text from the passages that there is not sufficient original work to be scored.

*Students are responsible for language conventions learned in their current grade as well as in prior grades. Refer to the language skills for each grade to determine the grade-level expectations for grammar, syntax, capitalization, punctuation, and spelling. Also refer to the Progressive Skills chart for those standards that need continued attention beyond the grade in which they were introduced.

Page 43: Georgia Milestones Grade 6 EOG Assessment Guide

Georgia Milestones Grade 6 EOG Assessment Guide | English Language Arts (ELA)

Georgia Department of Education November 2014 Page 42 of 101

All rights reserved.

Seven Point Two-Trait Rubric

Trait 1 for Informational/Explanatory Genre

Description Points Criteria

Idea Development, Organization, and Coherence This trait contributes 4 of 7 points to the score for this genre and examines the writer’s ability to effectively establish a controlling topic and to support the topic with evidence from the text(s) read and to elaborate on the topic with examples, illustrations, facts, and other details. The writer must integrate the information from the text(s) into his/her own words and arrange the ideas and supporting evidence in order to create cohesion for an informative/explanatory essay.

4 The student’s response is a well-developed informative/explanatory text that effectively examines a topic in depth and presents information clearly based on text as a stimulus.

Effectively introduces a topic

Effectively organizes ideas, concepts and information using various strategies such as definition, classification, comparison/contrast, and cause/effect

Effectively develops a topic with multiple, relevant facts, definitions, concrete details, quotations, or other information and examples

Uses transitions effectively to connect and clarify relationships among ideas

Uses precise language and domain-specific vocabulary to effectively inform and explain about the topic

Establishes and maintains formal style

Provides a strong concluding statement or section that logically follows from the information or explanation presented

3 The student’s response is a complete informative/explanatory text that examines a topic and presents information clearly based on text as a stimulus.

Introduces a topic

Generally organizes ideas, concepts and information using a few strategies such as definition, classification, comparison/contrast, and cause/effect

Develops a topic with a few facts, definitions, concrete details, quotations, or other information and examples

Uses some transitions to connect and clarify relationships among ideas, but relationships may not always be clear

Uses some precise language and domain-specific vocabulary to inform and explain about the topic

Uses formal style

Provides a concluding statement or section that follows from the information or explanation presented

2 The student’s response is an incomplete or oversimplified informative/explanatory text that that cursorily examines a topic.

Attempts to introduce a topic

Ineffectively organizes ideas, concepts and information using strategies such as definition, classification, comparison/contrast, and cause/effect

Attempts to develop a topic with too few details

Uses limited language and vocabulary that does not inform or explain the topic

Uses few transitions to connect and clarify relationships among ideas.

Uses formal style inconsistently or uses an informal style

Provides a weak concluding statement or section

1 The student’s response is a weak attempt to write an informative/explanatory text that examines a topic.

May not introduce a topic or topic is unclear

May not organize ideas

May not develop a topic with details

Uses vague, ambiguous, or repetitive language

May be too brief to connect related ideas

Uses informal style

Provides a minimal or no concluding statement or section

0 The response is completely irrelevant or incorrect, or there is no response.

The student merely copies the text in the prompt.

The student copies so much text from the passages that there is not sufficient original work to be scored.

Page 44: Georgia Milestones Grade 6 EOG Assessment Guide

Georgia Milestones Grade 6 EOG Assessment Guide | English Language Arts (ELA)

Georgia Department of Education November 2014 Page 43 of 101

All rights reserved.

Seven Point Two-Trait Rubric

Trait 2 for Informational/Explanatory Genres

Description Points Criteria

Language Usage and

Conventions

This trait contributes 3 of 7

points for this genre and

examines the writer’s ability

to demonstrate control of

sentence formation, usage,

and mechanics as embodied

in the grade-level

expectations of the

language standards.

3 The student’s response demonstrates full command of language usage and conventions.

Effectively varies sentence patterns for meaning, reader/listener interest, and style

Maintains consistency in style and tone

Shows command of language and conventions when writing

Any errors in usage and conventions do not interfere with meaning*

2 The student’s response demonstrates partial command of language usage and conventions.

Varies some sentence patterns for meaning, reader/listener interest and style

Generally maintains consistent style and tone

Shows some knowledge of languages and conventions when writing

Has minor errors in usage and conventions with no significant effect on meaning*

1 The student’s response demonstrates weak command of language usage and conventions.

Uses simple sentence patterns with little variety

Shows inconsistency in style and tone

Shows little knowledge of languages and conventions when writing

Has frequent errors in usage and conventions that interfere with meaning*

0 The student's response has many errors that affect the overall meaning, or the response is too brief to determine a score.

The student copies so much text from the passages that there is not sufficient original work to be scored.

*Students are responsible for language conventions learned in their current grade as well as in prior grades. Refer to the language skills for each grade to determine the grade-level expectations for grammar, syntax, capitalization, punctuation, and spelling. Also refer to the Progressive Skills chart for those standards that need continued attention beyond the grade in which they were introduced.

Page 45: Georgia Milestones Grade 6 EOG Assessment Guide

Georgia Milestones Grade 6 EOG Assessment Guide | English Language Arts (ELA)

Georgia Department of Education November 2014 Page 44 of 101

All rights reserved.

Seven Point Two-Trait Rubric

Trait 1 for Argumentative Genre

Description Points Criteria

Idea Development, Organization, and Coherence This trait contributes 4 of 7 points to the score for this genre and examines the writer’s ability to effectively establish a claim as well as to address counterclaims, to support the claim with evidence from the text(s) read, and to elaborate on the claim with examples, illustrations, facts, and other details. The writer must integrate the information from the text(s) into his/her own words and arrange the ideas and supporting evidence in order to create cohesion for an argumentative essay.

4 The student’s response is a well-developed argument that effectively relates and supports claims with clear reasons and relevant evidence.

Effectively introduces an opinion or claim and organizes supporting reasons and evidence clearly

Supports claims with clear reasons and relevant evidence using specific, well-chosen facts, details, or other information from credible sources

Uses words, phrases, or clauses effectively to connect ideas and clarify relationships among claims and reasons

Establishes and maintains formal style that is appropriate for the task, purpose, and audience

Provides a strong concluding statement or section that logically follows from the argument presented

3 The student’s response is a complete argument that relates and supports claims with some evidence.

Introduces a claim or opinion and organizes supporting reasons

Supports claims with reasons and evidence using some facts, details, or other information from generally credible sources

Uses words, phrases, or clauses to connect ideas and link claims and reasons

Uses formal style fairly consistently for the task, purpose, and audience

Provides a concluding statement or section that follows from the argument presented

2 The student’s response is an incomplete or oversimplified argument that relates and partially supports claims with loosely related evidence.

Attempts to introduce a claim or opinion

Attempts to support claims and opinions with reasons and evidence sometimes, but relevancy is often unclear

Uses few words, phrases, or clauses to connect ideas and link claims and reasons.

Uses formal style inconsistently or uses informal style

Provides a weak concluding statement or section

1 The student’s response is a weak attempt to write an argument and does not support claims with adequate evidence.

May not introduce a claim or opinion or claim

May not support claims or opinions

May be too brief to connect related ideas

Uses very informal style that is not appropriate for task, purpose, or audience

Provides a minimal or no concluding statement or section

0 The response is completely irrelevant or incorrect, or there is no response.

The student merely copies the text in the prompt.

The student copies so much text from the passages that there is not sufficient original work to be scored.

Page 46: Georgia Milestones Grade 6 EOG Assessment Guide

Georgia Milestones Grade 6 EOG Assessment Guide | English Language Arts (ELA)

Georgia Department of Education November 2014 Page 45 of 101

All rights reserved.

Seven Point Two-Trait Rubric

Trait 2 for Argumentative Genre

Description Points Criteria

Language Usage and

Conventions

This trait contributes 3 of 7

points for this genre and

examines the writer’s ability

to demonstrate control of

sentence formation, usage,

and mechanics as embodied

in the grade-level

expectations of the

language standards.

3 The student’s response demonstrates full command of language usage and conventions.

Effectively varies sentence patterns for meaning, reader/listener interest, and style

Maintains consistency in style and tone

Shows command of language and conventions when writing

Any errors in usage and conventions do not interfere with meaning*

2 The student’s response demonstrates partial command of language usage and conventions.

Varies some sentence patterns for meaning, reader/listener interest and style

Generally maintains consistent style and tone

Shows some knowledge of languages and conventions when writing

Has minor errors in usage and conventions with no significant effect on meaning*

1 The student’s response demonstrates weak command of language usage and conventions.

Uses simple sentence patterns with little variety

Shows inconsistency in style and tone

Shows little knowledge of languages and conventions when writing

Has frequent errors in usage and conventions that interfere with meaning*

0 The student's response has many errors that affect the overall meaning, or the response is too brief to determine a score.

The student copies so much text from the passages that there is not sufficient original work to be scored.

*Students are responsible for language conventions learned in their current grade as well as in prior grades. Refer to the language skills for each grade to determine the grade-level expectations for grammar, syntax, capitalization, punctuation, and spelling. Also refer to the Progressive Skills chart for those standards that need continued attention beyond the grade in which they were introduced.

Page 47: Georgia Milestones Grade 6 EOG Assessment Guide

Georgia Milestones Grade 6 EOG Assessment Guide | Mathematics

Georgia Department of Education November 2014 Page 46 of 101

All rights reserved.

MATHEMATICS

Description of Test Format and Organization

The Georgia Milestones EOG assessment is primarily a criterion-referenced test, designed to provide

information about how well a student has mastered the grade-level state-adopted content standards in

Mathematics. Each student will receive one of four proficiency levels, depending on how well the

student has mastered the content standards. In addition to criterion-referenced information, the

Georgia Milestones measures will also include a limited sample of nationally norm-referenced items to

provide a signal of how Georgia students are achieving relative to their peers nationally. The norm-

referenced information provided is supplementary to the criterion-referenced proficiency designation

and will not be utilized in any manner other than to serve as a barometer of national comparison. Only

the criterion-referenced scores and proficiency designations will be utilized in the accountability metrics

associated with the assessment program (such as student growth measures, educator effectiveness

measures, or the CCRPI).

The Grade 6 Mathematics EOG assessment consists of a total of 73 items, 64 of which are operational

items (and contribute to a student’s criterion-referenced and/or norm-referenced score) and 9 of which

are field test items (newly written items that are being tried out and do not contribute to the student’s

score). The criterion-referenced score, and proficiency designation, is comprised of 53 items, for a total

of 58 points. Students will respond to a variety of item types, including selected-response, constructed-

response, and extended constructed-response items. Of the 64 operational items, 20 will be norm-

referenced and will provide a national comparison in the form of a national percentile rank. Nine of the

items have been verified as aligned to the course content standards by Georgia educators and will

therefore contribute to the criterion-referenced proficiency designation. The other 11 items will

contribute only to the national percentile rank and be provided as supplemental information. Only items

that are aligned to the state-adopted content standards will be utilized to inform the criterion-

referenced score.

With the inclusion of the norm-referenced items, students may encounter items for which they have not

received direct instruction. These items will not contribute to the student’s criterion-referenced

proficiency designation; only items that align to the course content standards will contribute to the

criterion-referenced score. Students should be instructed to try their best should they ask about an item

that is not aligned to the content they have learned as part of the course.

Page 48: Georgia Milestones Grade 6 EOG Assessment Guide

Georgia Milestones Grade 6 EOG Assessment Guide | Mathematics

Georgia Department of Education November 2014 Page 47 of 101

All rights reserved.

Grade 6 Mathematics EOG Assessment Design

Description Number of

Items Points for CR1 Score

Points for NRT2 Feedback

CR Selected-Response Items 41 41 0

NRT Selected-Response Items 203 94 20

CR Constructed-Response Items 3 8 0

CR Field Test Items 9 0 0

Total Items/Points5 73 58 20 1CR—Criterion-Referenced: items aligned to state-adopted content standards 2NRT—Norm-Referenced Test: items that will yield a national comparison; may or may not be aligned to state-adopted content standards 3Of these items, 9 will contribute to both the CR scores and NRT feedback. The other 11 of these items will contribute to NRT feedback only and will not impact the student’s proficiency designation, scale score, or grade conversion. 4Alignment of national NRT items to course content standards was verified by a committee of Georgia educators. Only approved, aligned NRT items will contribute to a student’s CR proficiency designation, scale score, and grade conversion score. 5Total number of items contributing to CR score: 53; total points: 58; total number of items contributing to NRT feedback: 20; total points: 20

The test will be given in two sections. Section 1 is divided into two parts. Students may have up to 80

minutes per section to complete Sections 1 and 2. The total estimated testing time for the Grade 6

Mathematics EOG assessment ranges from approximately 120 to 160 minutes. Total testing time

describes the amount of time students have to complete the assessment. It does not take into account

the time required for the test examiner to complete pre-administration and post-administration

activities (such as reading the standardized directions to students). Sections 1 and 2 must be scheduled

to be administered on the same day in one test session following the district’s testing protocols for the

EOG measures (in keeping with state guidance).

During the Mathematics EOG assessment, a formula sheet will be available for students to use. There is

an example of the formula sheet in the Additional Sample Items section of this guide. Another feature of

the Grade 6 Mathematics EOG assessment is that students may use a basic function calculator in Part 1

of Section 1 and in all of Section 2.

Content Measured

The Grade 6 Mathematics assessment will measure the standards that are enumerated for Grade 6 as

described on www.georgiastandards.org.

The content of the assessment is organized into five groupings, or domains, of standards for the

purposes of providing feedback on student performance. A content domain is a reporting category that

broadly describes and defines the content of the course, as measured by the EOG assessment. The

standards for Grade 6 Mathematics are grouped into five domains: Ratios and Proportional

Relationships, The Number System, Expressions and Equations, Geometry, and Statistics and Probability.

Each domain was created by organizing standards that share similar content characteristics. The content

Page 49: Georgia Milestones Grade 6 EOG Assessment Guide

Georgia Milestones Grade 6 EOG Assessment Guide | Mathematics

Georgia Department of Education November 2014 Page 48 of 101

All rights reserved.

standards describe the level of expertise that Grade 6 Mathematics educators should strive to develop

in their students. Educators should refer to the content standards for a full understanding of the

knowledge, concepts, and skills subject to be assessed on the EOG assessment.

The approximate proportional number of points associated with each domain is shown in the following

table. A range of cognitive levels will be represented on the Grade 6 Mathematic s EOG assessment.

Educators should always use the content standards when planning instruction.

Grade 6 Mathematics: Domain Structures and Content Weights

Domain Standard Approximate

Weight

Ratios and Proportional Relationships

MCC6RP1 MCC6RP2 MCC6RP3

12%

The Number System

MCC6NS1 MCC6NS2 MCC6NS3 MCC6NS4

MCC6NS5 MCC6NS6 MCC6NS7 MCC6NS8

30%

Expressions and Equations

MCC6EE1 MCC6EE2 MCC6EE3 MCC6EE4 MCC6EE5

MCC6EE6 MCC6EE7 MCC6EE8 MCC6EE9

23%

Geometry

MCC6G1 MCC6G2 MCC6G3 MCC6G4

18%

Statistics and Probability MCC6SP1 MCC6SP2 MCC6SP3

MCC6SP4 MCC6SP5

17%

Page 50: Georgia Milestones Grade 6 EOG Assessment Guide

Georgia Milestones Grade 6 EOG Assessment Guide | Mathematics

Georgia Department of Education November 2014 Page 49 of 101

All rights reserved.

Item Types

The Mathematics portion of the Grade 6 EOG assessment consists of selected-response, constructed-

response, and extended constructed-response items.

A selected-response item, sometimes called a multiple-choice item, is defined as a question, problem, or

statement that appears on a test followed by several answer choices, sometimes called options or

response choices. The incorrect choices, called distractors, usually reflect common errors. The student’s

task is to choose, from the alternatives provided, the best answer to the question posed in the stem (the

question). The Mathematics selected-response items will have four answer choices.

A constructed-response item asks a question and solicits the student to provide a response he or she

constructs on his or her own, as opposed to selecting from options provided. The constructed-response

items on the EOG assessment will be worth two points. Partial credit may be awarded.

An extended constructed-response item is a specific type of constructed-response item that elicits a

longer, more detailed response from the student than a two-point constructed-response item. The

extended constructed-response items on the EOG assessment will be worth four points. Partial credit

may be awarded.

Mathematics Example Items

Example items, which are representative of three DOK levels across various Grade 6 Mathematics

content domains, are provided on the following pages. All example and sample items contained in this

guide are the property of the Georgia Department of Education.

Page 51: Georgia Milestones Grade 6 EOG Assessment Guide

Georgia Milestones Grade 6 EOG Assessment Guide | Mathematics

Georgia Department of Education November 2014 Page 50 of 101

All rights reserved.

Example Item 1

DOK Level: 1

Mathematics Grade 6 Content Domain: The Number System

Standard: MCC6NS6c. Find and position integers and other rational numbers on a horizontal or vertical

number line diagram; find and position pairs of integers and other rational numbers on a coordinate

plane.

Look at point P on the coordinate grid.

What are the coordinates of point P?

A (2, 4) B (4, 2) C (–2, –4) D (–4, –2)

Correct Answer: C

Explanation of Correct Answer: The correct answer is choice (C) (–2, –4). Point P is located 2 units to the

left of the origin which gives us a value of –2 for x, and 4 units down which gives us a value of –4 for y.

Choice (A) is incorrect because the signs of the numbers are ignored. Choice (B) is incorrect because the

coordinates are interchanged and the signs are ignored. Choice (D) is incorrect because it interchanges

the coordinates.

Page 52: Georgia Milestones Grade 6 EOG Assessment Guide

Georgia Milestones Grade 6 EOG Assessment Guide | Mathematics

Georgia Department of Education November 2014 Page 51 of 101

All rights reserved.

Example Item 2

DOK Level: 2

Mathematics Grade 6 Content Domain: Ratios and Proportional Relationships

Standard: MCC6RP3. Use ratio and rate reasoning to solve real-world and mathematical problems, e.g.,

by reasoning about tables of equivalent ratios, tape diagrams, double number line diagrams, or

equations. b. Solve unit rate problems including those involving unit pricing and constant speed. For

example, if it took 7 hours to mow 4 lawns, then at that rate, how many lawns could be mowed in 35

hours? At what rate were lawns being mowed?

John orders 25 prints from a photo store for $13.00.

What is the cost per print?

A $0.12 B $0.38 C $0.52 D $1.92

Correct Answer: C

Explanation of Correct Answer: The correct answer is choice (C) $0.52. The cost per print is equal to the

total cost divided by the number of prints: $13.00

25 = $0.52. Choice (A) is incorrect because it is the result

of subtracting 0.13 from 0.25. Choice (B) is incorrect because it is the result of adding 0.13 and 0.25.

Choice (D) is incorrect because it is the result of dividing 25 by 13.

Page 53: Georgia Milestones Grade 6 EOG Assessment Guide

Georgia Milestones Grade 6 EOG Assessment Guide | Mathematics

Georgia Department of Education November 2014 Page 52 of 101

All rights reserved.

Example Item 3

DOK Level: 3

Mathematics Grade 6 Content Domain: Geometry

Standard: MCC6G3. Draw polygons in the coordinate plane given coordinates for the vertices; use

coordinates to find the length of a side joining points with the same first coordinate or the same second

coordinate. Apply these techniques in the context of solving real-world and mathematical problems.

Harry is drawing trapezoid PQRS. He plots vertices P and Q on the coordinate grid as shown.

Harry wants the trapezoid to have a height of 3 units.

Which of these could be the coordinates of vertices R and S of trapezoid PQRS?

A R(2, 3) and S(–3, 3) B R(3, –3) and S(–4, –3) C R(4, –2) and S(–2, –2) D R(–2, 4) and S(2, 2)

Correct Answer: C

Explanation of Correct Answer: The correct answer is choice (C) R(4, –2) and S(–2, –2). The given

coordinates form a trapezoid. From –2 to 1 on the y-axis is a height of 3 units. While choices (A) and (B)

do give coordinates that form trapezoids, the heights are not 3 units. In choice (A), the height is 2 units.

In choice (B), the height is 4 units. Choice (D) is incorrect because the given coordinates do not form a

trapezoid.

Page 54: Georgia Milestones Grade 6 EOG Assessment Guide

Georgia Milestones Grade 6 EOG Assessment Guide | Mathematics

Georgia Department of Education November 2014 Page 53 of 101

All rights reserved.

Mathematics Additional Sample Items

This section has two parts. The first part is a set of 10 sample items for the Mathematics portion of the

EOG assessment. The second part contains a table that shows for each item the standard assessed, the

DOK level, the correct answer (key), and a rationale/explanation about the key and distractors. The

sample items can be utilized as a mini-test to familiarize students with the item formats found on the

assessment. All example and sample items contained in this guide are the property of the Georgia

Department of Education.

Page 55: Georgia Milestones Grade 6 EOG Assessment Guide

Georgia Milestones Grade 6 EOG Assessment Guide | Mathematics

Georgia Department of Education November 2014 Page 54 of 101

All rights reserved.

Item 1

Look at the expression.

25 + 45

Which of these is an equivalent expression?

A 5(5 + 45) B 5(5 + 9) C 5(20 + 40) D 5(25 + 9)

Item 2

Which inequality is true and why?

A –1 > –4 because –1 is to the right of –4 on a horizontal number line oriented left to right B –4 > –5 because –4 is to the left of –5 on a horizontal number line oriented left to right C –8 > –7 because –8 is to the right of –7 on a horizontal number line oriented left to right D –9 > –6 because –9 is to the left of –6 on a horizontal number line oriented left to right

Page 56: Georgia Milestones Grade 6 EOG Assessment Guide

Georgia Milestones Grade 6 EOG Assessment Guide | Mathematics

Georgia Department of Education November 2014 Page 55 of 101

All rights reserved.

Item 3

Look at the figure.

What is the total area of this figure?

A 141 ft2 B 171 ft2 C 180 ft2 D 195 ft2

Page 57: Georgia Milestones Grade 6 EOG Assessment Guide

Georgia Milestones Grade 6 EOG Assessment Guide | Mathematics

Georgia Department of Education November 2014 Page 56 of 101

All rights reserved.

Item 4

Faye made a case for her electronic reading device using the net shown.

What is the total surface area, in square inches, of Faye’s case?

A 62 B 96 C 108 D 124

Page 58: Georgia Milestones Grade 6 EOG Assessment Guide

Georgia Milestones Grade 6 EOG Assessment Guide | Mathematics

Georgia Department of Education November 2014 Page 57 of 101

All rights reserved.

Item 5

Look at the box in the shape of a right rectangular prism.

Lorraine plans to fill this box with cubes of the same size. Each cube has side lengths of 𝟏

𝟐 inch.

How many cubes can fit inside this box without empty space?

A 4 B 5 C 32 D 60

Page 59: Georgia Milestones Grade 6 EOG Assessment Guide

Georgia Milestones Grade 6 EOG Assessment Guide | Mathematics

Georgia Department of Education November 2014 Page 58 of 101

All rights reserved.

Item 6

The line plot shows the ages of the children who had their photographs taken at a photography studio

during a certain week.

Which statement about the children who had their photographs taken does the spread of the data

describe?

A The average age of the children was 3 years. B The most common age of the children was 10 years. C The ages of half of the children were 6 years or less. D The ages of the children ranged from 1 year to 10 years.

Item 7

Look at the inequality.

3y > 27

Which set of values for y will make this inequality true?

A 4, 5, 8 B 5, 7, 9 C 9, 12, 14 D 11, 13, 22

Page 60: Georgia Milestones Grade 6 EOG Assessment Guide

Georgia Milestones Grade 6 EOG Assessment Guide | Mathematics

Georgia Department of Education November 2014 Page 59 of 101

All rights reserved.

Item 8

Harriet has $25 in her savings account. She will add $5 each week and not take any money out of her

account.

Which expression represents the amount of money in her savings account after x weeks?

A 25x + 5 B 5x + 25 C 25(x + 5) D 5(x + 25)

Item 9

Mary has 𝟐

𝟑 cup of chocolate chips. A cookie recipe calls for

𝟏

𝟐 cup of chocolate chips per batch.

How many batches can Mary make using all of her chocolate chips? Show your work and write your

answer in the space provided.

__________ batches

Page 61: Georgia Milestones Grade 6 EOG Assessment Guide

Georgia Milestones Grade 6 EOG Assessment Guide | Mathematics

Georgia Department of Education November 2014 Page 60 of 101

All rights reserved.

Item 10

Mack's Market and Gary's Gardens each sell watermelons by the pound. At Gary’s Gardens, a 3-pound

watermelon costs $1.26. At Mack’s Market, a 5-pound watermelon costs $2.25.

Part A: What is the cost per pound of a watermelon at Gary’s Gardens?

____________________

Part B: Knowing that the unit price remains the same, how much would a 6-pound watermelon cost at

Mack’s Market?

____________________

Part C: Kate wants to buy a 12-pound watermelon for the lowest price.

How much would she save by buying the watermelon at Gary's Gardens? Show your work and write

your answer in the space provided.

__________

Page 62: Georgia Milestones Grade 6 EOG Assessment Guide

Georgia Milestones Grade 6 EOG Assessment Guide | Mathematics

Georgia Department of Education November 2014 Page 61 of 101

All rights reserved.

Mathematics Additional Sample Item Keys

Item Standard/ Element

DOK Level Correct Answer

Explanation

1 6NS4 2 B

The correct answer is choice (B) 5(5 + 9). A

common factor of 25 and 45 is 5. 25

5 = 5 and

45

5 =

9. So, 25 + 45 = 5(5 + 9). Choices (A) and (D) are incorrect because they only factor one of the two terms. Choice (C) is incorrect because it is the result of subtracting 5 from each term instead of dividing by 5.

2 6NS7 2 A

The correct answer is choice (A) –1 > –4 because –1 is to the right of –4 on a horizontal number line oriented left to right. Numbers increase in value moving from left to right along the number line. Since –1 is to the right of –4 on the number line, –1 > –4. Choice (B) is incorrect because it confuses the location of the numbers. Choice (C) is incorrect because it does not consider the signs of the numbers. Choice (D) is incorrect because it misstates the relative positions of the numbers on the number line.

3 6G1 2 B

The correct answer is choice (B) 171 ft2. The area of the smaller rectangle is 3 x 9 = 27 square feet. The area of the larger rectangle is 10 x 12 = 120 square feet. The area of the

triangle is (1

2)(12 x 4) = 24 square feet. The total

area is 27 + 120 + 24 = 171 square feet. Choice (A) is incorrect because it uses a width of 9 feet for the larger rectangle, instead of 12 feet. Choice (C) is incorrect because it combines the two rectangles into one rectangle with dimensions 12 feet by 13 feet. Choice (D) is incorrect because it uses 4 x 12 as the area of

the triangle instead of (1

2)(12 x 4).

4 6G4 2 D

The correct answer is choice (D) 124. The net is comprised of two rectangles each measuring 8 x 6 inches, two rectangles each measuring 1 x 8 inches, and two rectangles each measuring 1 x 6 inches. The total area is 2(8 x 6) + 2(1 x 8) + 2(1 x 6) = 124 square inches. Choices (A), (B), and (C) are incorrect because they do not include all of the faces of the net.

Page 63: Georgia Milestones Grade 6 EOG Assessment Guide

Georgia Milestones Grade 6 EOG Assessment Guide | Mathematics

Georgia Department of Education November 2014 Page 62 of 101

All rights reserved.

Item Standard/ Element

DOK Level Correct Answer

Explanation

5 6G2 2 D

The correct answer is choice (D) 60. Along the length of the box, 5 cubes will fit. Along the width, 3 cubes will fit. Therefore, 15 cubes will fill the base. The box will hold 4 layers of cubes. That makes the total number of cubes 60. Choices (A) and (B) are incorrect because they are based on the base holding 2 x 1 cubes and there being only 2 layers of cubes. Choice (B)

then adds 1 more cube to account for the 1

2 in

the given length and width of the box. Choice (C) is incorrect because it is based on the box holding 4 x 2 cubes in the base or 32 cubes instead of 15.

6 6SP2 1 D

The correct answer is choice (D) The ages of the children ranged from 1 year to 10 years. The least number on the number line with Xs above it is 1. The greatest number on the number line with Xs above it is 10. The data values range from 1 to 10. Choices (A) and (B) are incorrect because they are statements about measures of center instead of a measure of spread. Choice (C) is incorrect because it assumes the spread of the data is the number of ages with Xs above them.

7 6EE5 2 D

The correct answer is choice (D) 11, 13, 22. The values of y that will make the inequality true are the values of y for which 3y is greater than 27. 3(11) = 33; 3(13) = 39; 3(22) = 66. Choices (A) and (B) are incorrect because at least one value of y makes 3y less than 27. Choice (C) is incorrect because when y is 9, 3y is equal to 27.

8 6EE6 2 B

The correct answer is (B) 5x + 25. Harriet starts with $25, so the expression must have a value of 25 when x =0. Each week, Harriet adds $5, so the rate of change is $5 per week. Choice (A) is incorrect because it interchanges the starting amount with the amount added each week. Choices (C) and (D) use the distributive property incorrectly for this situation.

9 6NS1 2 N/A See scoring rubric and exemplar responses beginning on page 63.

10 6RP3a 3 N/A See scoring rubric and exemplar responses beginning on page 64.

Page 64: Georgia Milestones Grade 6 EOG Assessment Guide

Georgia Milestones Grade 6 EOG Assessment Guide | Mathematics

Georgia Department of Education November 2014 Page 63 of 101

All rights reserved.

Mathematics Example Scoring Rubrics and Exemplar Responses

Item 9

Scoring Rubric

Points Description

2

The response achieves the following:

The response demonstrates a complete understanding of interpreting and

computing quotients of fractions and solving word problems involving

division of fractions by fractions.

Give 2 points for a correct process AND the correct answer.

o Response is correct and complete.

o Response shows application of a reasonable and relevant strategy.

Mathematical ideas are expressed coherently through a complete, logical, and fully developed response using words, calculations, and/or symbols as appropriate.

1

The response achieves the following:

The response demonstrates a partial understanding of interpreting and

computing quotients of fractions and solving word problems involving

division of fractions by fractions.

Give 1 point for a correct process OR a correct answer with no work shown.

o Response is only partially correct.

o Response shows application of a relevant strategy, though it may

be only partially applied or remain unexplained.

Mathematical ideas are expressed only partially using words, calculations, and/or symbols as appropriate.

0

The response achieves the following:

The response demonstrates no understanding of interpreting and

computing quotients of fractions and solving word problems involving

division of fractions by fractions.

o Response shows no application of a strategy or application of an

irrelevant strategy.

Mathematical ideas cannot be interpreted or lack sufficient evidence to support even a limited understanding.

Exemplar Response

Points Awarded

Response

2

2

3÷1

2=2

3×2

1=4

3= 1

1

3

11

3 batches

Page 65: Georgia Milestones Grade 6 EOG Assessment Guide

Georgia Milestones Grade 6 EOG Assessment Guide | Mathematics

Georgia Department of Education November 2014 Page 64 of 101

All rights reserved.

Item 9

Exemplar Response – continued

Points Awarded

Response

1 11

3 batches

0 Response is irrelevant, inappropriate, or not provided.

Item 10

Scoring Rubric

Points Description

4

The response achieves the following:

The response demonstrates a complete understanding of using ratio and

rate reasoning to solve real-world and mathematical problems.

Give 4 points for Part A and Part B and Part C completely correct.

o Response is correct and complete.

o Response shows application of a reasonable and relevant strategy.

Mathematical ideas are expressed coherently through a clear, complete, logical, and fully developed response using words, calculations, and/or symbols as appropriate.

3

The response achieves the following:

The response demonstrates a good understanding of using ratio and rate

reasoning to solve real-world and mathematical problems.

Give 3 points for Part A and Part B correct and Part C partially correct OR

Part A or Part B correct and Part C correct based on error in previous parts.

o Response is mostly correct, but contains either a computation error

or an unclear or incomplete explanation.

o Response shows application of a relevant strategy, though it may

be only partially applied or remain unexplained.

Mathematical ideas are expressed only partially using words, calculations, and/or symbols as appropriate.

2

The response achieves the following:

The response demonstrates a partial understanding of using ratio and rate

reasoning to solve real-world and mathematical problems.

Give 2 points for Part A and Part B correct OR for Part C correct based on

incorrect answers given in Part A and Part B.

o Response is only partially correct.

o Response shows application of a relevant strategy, though it may

be only partially applied or remain unexplained.

Mathematical ideas are expressed only partially using words, calculations, and/or symbols as appropriate.

Page 66: Georgia Milestones Grade 6 EOG Assessment Guide

Georgia Milestones Grade 6 EOG Assessment Guide | Mathematics

Georgia Department of Education November 2014 Page 65 of 101

All rights reserved.

Item 10

Scoring Rubric – continued

Points Description

1

The response achieves the following:

The response demonstrates a limited understanding of using ratio and rate

reasoning to solve real-world and mathematical problems.

Give 1 point for Part A correct OR Part B correct OR Part C partially correct.

o Response is only partially correct.

o Response shows incomplete or inaccurate application of a relevant

strategy.

Mathematical ideas are expressed only partially using words, calculations, and/or symbols as appropriate.

0

The response achieves the following:

The response demonstrates no understanding of using ratio and rate

reasoning to solve real-world and mathematical problems.

o Response shows no application of a strategy.

Mathematical ideas cannot be interpreted or lack sufficient evidence to support even a limited understanding.

Exemplar Response

Points Awarded

Response

4

Part A: $0.42 Part B: $2.70 Part C: By shopping at Gary's Gardens, Kate will save $0.36 AND The difference in cost per pound at the two places is $0.03 per pound times 12 pounds, which is $0.36. OR other valid response

3

Part A: $0.42 Part B: $2.70 Part C: $0.36 OR other valid response

Page 67: Georgia Milestones Grade 6 EOG Assessment Guide

Georgia Milestones Grade 6 EOG Assessment Guide | Mathematics

Georgia Department of Education November 2014 Page 66 of 101

All rights reserved.

Item 10

Exemplar Response – continued

Points Awarded

Response

2 Part A: $0.42 Part B: $2.70

1 Part A: $0.42

0 Response is irrelevant, inappropriate, or not provided.

Page 68: Georgia Milestones Grade 6 EOG Assessment Guide

Georgia Milestones Grade 6 EOG Assessment Guide | Science

Georgia Department of Education November 2014 Page 67 of 101

All rights reserved.

SCIENCE

Description of Test Format and Organization

The Georgia Milestones EOG assessment is primarily a criterion-referenced test, designed to provide

information about how well a student has mastered the grade-level state-adopted content standards in

Science. Each student will receive one of four proficiency levels, depending on how well the student has

mastered the content standards. In addition to criterion-referenced information, the Georgia Milestones

measures will also include a limited sample of nationally norm-referenced items to provide a signal of

how Georgia students are achieving relative to their peers nationally. The norm-referenced information

provided is supplementary to the criterion-referenced proficiency designation and will not be utilized in

any manner other than to serve as a barometer of national comparison. Only the criterion-referenced

scores and proficiency designations will be utilized in the accountability metrics associated with the

assessment program (such as student growth measures, educator effectiveness measures, or the CCRPI).

The Grade 6 Science EOG assessment consists of a total of 75 selected-response items, 65 of which are

operational items (and contribute to a student’s criterion-referenced and/or norm-referenced score)

and 10 of which are field test items (newly written items that are being tried out and do not contribute

to the student’s score). The criterion-referenced score, and proficiency designation, is comprised of 55

items, for a total of 55 points. Of the 65 operational items, 20 will be norm-referenced and will provide a

national comparison in the form of a national percentile rank. Ten of the items have been verified as

aligned to the course content standards by Georgia educators and will therefore contribute to the

criterion-referenced proficiency designation. The other 10 items will contribute only to the national

percentile rank and be provided as supplemental information. Only items that are aligned to the state-

adopted content standards will be utilized to inform the criterion-referenced score.

With the inclusion of the norm-referenced items, students may encounter items for which they have not

received direct instruction. These items will not contribute to the student’s criterion-referenced

proficiency designation; only items that align to the course content standards will contribute to the

criterion-referenced score. Students should be instructed to try their best should they ask about an item

that is not aligned to the content they have learned as part of the course.

Page 69: Georgia Milestones Grade 6 EOG Assessment Guide

Georgia Milestones Grade 6 EOG Assessment Guide | Science

Georgia Department of Education November 2014 Page 68 of 101

All rights reserved.

Grade 6 Science EOG Assessment Design

Description Number of

Items Points for CR1 Score

Points for NRT2 Feedback

CR Selected-Response Items 45 45 0

NRT Selected-Response Items 203 104 20

CR Field Test Items 10 0 0

Total Items/Points5 75 55 20 1CR—Criterion-Referenced: items aligned to state-adopted content standards 2NRT—Norm-Referenced Test: items that will yield a national comparison; may or may not be aligned to state-adopted content standards 3Of these items, 10 will contribute to both the CR scores and NRT feedback. The other 10 of these items will contribute to NRT feedback only and will not impact the student’s proficiency designation, scale score, or grade conversion. 4Alignment of national NRT items to course content standards was verified by a committee of Georgia educators. Only approved, aligned NRT items will contribute to a student’s CR proficiency designation, scale score, and grade conversion score. 5Total number of items contributing to CR score: 55; total points: 55; total number of items contributing to NRT feedback: 20; total points: 20

The test will be given in two sections. Students may have up to 70 minutes per section to complete

Sections 1 and 2. The total estimated testing time for the Grade 6 Science EOG assessment ranges from

approximately 100 to 140 minutes. Total testing time describes the amount of time students have to

complete the assessment. It does not take into account the time required for the test examiner to

complete pre-administration and post-administration activities (such as reading the standardized

directions to students). Sections 1 and 2 must be scheduled to be administered on the same day in one

test session following the district’s testing protocols for the EOG measures (in keeping with state

guidance).

Content Measured

The Grade 6 Science assessment will measure the standards that are enumerated for Grade 6 as

described on www.georgiastandards.org.

The content of the assessment is organized into three groupings, or domains, of standards for the

purposes of providing feedback on student performance. A content domain is a reporting category that

broadly describes and defines the content of the course, as measured by the EOG assessment. The

standards for Grade 6 Science are grouped into three domains: Astronomy, Geology, and Hydrology and

Meteorology. Each domain was created by organizing standards that share similar content

characteristics. The content standards describe the level of expertise that Grade 6 Science educators

should strive to develop in their students. Educators should refer to the content standards for a full

understanding of the knowledge, concepts, and skills subject to be assessed on the EOG assessment.

The approximate proportional number of points associated with each domain is shown in the following

table. A range of cognitive levels will be represented on the Grade 6 Science EOG assessment. Educators

should always use the content standards when planning instruction.

Page 70: Georgia Milestones Grade 6 EOG Assessment Guide

Georgia Milestones Grade 6 EOG Assessment Guide | Science

Georgia Department of Education November 2014 Page 69 of 101

All rights reserved.

Grade 6 Science: Domain Structures and Content Weights

Domain Standard Approximate

Weight

Astronomy

S6E1 (1a, 1b, 1c, 1d, 1e, 1f)

S6E2 (2a, 2b, 2c)

20%

Geology

S6E5 (5a, 5b, 5c, 5d, 5e, 5f, 5g, 5h, 5i, 5j)

S6E6 (6b)

40%

Hydrology and Meteorology

S6E3 (3a, 3b, 3c, 3d)

S6E4 (4a, 4b, 4c)

S6E6 (6a)

40%

Page 71: Georgia Milestones Grade 6 EOG Assessment Guide

Georgia Milestones Grade 6 EOG Assessment Guide | Science

Georgia Department of Education November 2014 Page 70 of 101

All rights reserved.

Item Types

The Science portion of the Grade 6 EOG assessment consists of selected-response items only.

A selected-response item, sometimes called a multiple-choice item, is defined as a question, problem, or

statement that appears on a test followed by several answer choices, sometimes called options or

response choices. The incorrect choices, called distractors, usually reflect common errors. The student’s

task is to choose, from the alternatives provided, the best answer to the question posed in the stem (the

question). The Science selected-response items will have four answer choices.

Science Example Items

Example items, which are representative of three DOK levels across various Grade 6 Science content

domains, are provided on the following pages. All example and sample items contained in this guide

are the property of the Georgia Department of Education.

Page 72: Georgia Milestones Grade 6 EOG Assessment Guide

Georgia Milestones Grade 6 EOG Assessment Guide | Science

Georgia Department of Education November 2014 Page 71 of 101

All rights reserved.

Example Item 1

DOK Level: 1

Science Grade 6 Content Domain: Hydrology and Meteorology

Standard: S6E6. Students will describe various sources of energy and with their uses and conservation.

a. Explain the role of the sun as the major source of energy and its relationship to wind and water

energy.

Which of these BEST explains how wind is formed?

A by the rotation of Earth on its axis B by the movement of tectonic plates C by the uneven heating of Earth's surface D by the gravitational pull between the Moon and Earth

Correct Answer: C

Explanation of Correct Answer: The correct answer is choice (C) by the uneven heating of Earth's

surface. Some parts of Earth's surface receive more direct sunlight. As air heats in these regions, it rises

and is replaced by cooler air from regions that receive less direct sunlight. Eventually, the heated air

cools and sinks, the cool air heats and rises, and the cycle repeats. These movements of air across

Earth's surface we call "wind." Choice (A) is incorrect because although Earth's rotation has an effect on

wind direction, its major effect is to cause different parts of the planet to experience day and night.

Choice (B) is incorrect because tectonic plates cause geologic features to form. Choice (D) is incorrect

because the gravitational pull between the Moon and Earth causes tides on Earth.

Page 73: Georgia Milestones Grade 6 EOG Assessment Guide

Georgia Milestones Grade 6 EOG Assessment Guide | Science

Georgia Department of Education November 2014 Page 72 of 101

All rights reserved.

Example Item 2

DOK Level: 2

Science Grade 6 Content Domain: Geology

Standard: S6E5. Students will investigate the scientific view of how the earth’s surface is formed. e.

Recognize that lithospheric plates constantly move and cause major geological events on the earth’s

surface.

Scientists record frequent earthquakes and observe many volcanoes in the highlighted area shown on

the map.

Why do so many earthquakes and volcanoes occur in that area?

A The area is made up of only one type of rock. B The air pressure is the lowest in these coastal areas. C Ocean waves are constantly hitting these coastal areas. D Tectonic plates in the area are colliding or moving apart.

Correct Answer: D

Explanation of Correct Answer: The correct answer is choice (D) Tectonic plates in the area are colliding

or moving apart. The highlighted area reveals the boundaries of many tectonic plates. As these plates

collide and move apart, they produce earthquakes and volcanic eruptions. Choice (A) is incorrect

because the highlighted area is made up of many types of rock. Choices (B) and (C) are incorrect because

air pressure and ocean waves are unrelated to earthquakes and volcanoes.

Page 74: Georgia Milestones Grade 6 EOG Assessment Guide

Georgia Milestones Grade 6 EOG Assessment Guide | Science

Georgia Department of Education November 2014 Page 73 of 101

All rights reserved.

Example Item 3

DOK Level: 2

Science Grade 6 Content Domain: Geology

Standard: S6E5. Students will investigate the scientific view of how the earth’s surface is formed. i.

Explain the effects of human activity on the erosion of the earth’s surface.

In order to expand his farmland, a farmer cut down the forested area surrounding his farmland. The

next year the farmland had poor-quality soil.

Which of these is the MOST LIKELY reason the quality of soil changed?

A The wind eroded the soil. B The farmland received less shade. C The farmland had too much topsoil. D The soil received less water from the forest.

Correct Answer: A

Explanation of Correct Answer: The correct answer is choice (A) The wind eroded the soil. The roots of

trees help to hold soil in place, and the trees themselves help to block the wind; both of these factors

prevent erosion that can degrade soil. Cutting down trees both loosens soil and exposes it to more wind.

Choice (B) is incorrect because shade is unrelated to soil quality. Choice (C) is incorrect because topsoil

contains many nutrients; a farm with lots of topsoil would likely have high-quality soil. Choice (D) is

incorrect because precipitation, not trees, is the primary source of water for soil.

Page 75: Georgia Milestones Grade 6 EOG Assessment Guide

Georgia Milestones Grade 6 EOG Assessment Guide | Science

Georgia Department of Education November 2014 Page 74 of 101

All rights reserved.

Example Item 4

DOK Level: 3

Science Grade 6 Content Domain: Astronomy

Standard: S6E1. Students will explore current scientific views of the universe and how those views

evolved. c. Compare and contrast the planets in terms of

Size relative to the earth

Surface and atmospheric features

Relative distance from the sun

Ability to support life

Two scientists are investigating the similarities between Earth and Mars. The first scientist states that

Mars could support life because land is required for a planet to support life. The second scientist

states that Mars is unlikely to support life because water is required for a planet to support life.

Which scientist is correct?

A The first scientist is correct because there are organisms on Earth that can survive only on land. B The second scientist is correct because the polar ice caps on Mars are able to provide water to the entire planet. C The first scientist is correct because the mountains and valleys on Mars provide a possible home for plants and animals. D The second scientist is correct because all organisms on Earth require liquid water to survive in their

specific environment.

Correct Answer: D

Explanation of Correct Answer: The correct answer is choice (D) The second scientist is correct because

all organisms on Earth require liquid water to survive in their specific environment. Although research

suggests that Mars may contain liquid water beneath its frozen surface, scientists have not found liquid

water on the planet's surface, making the planet unlikely to support life as we know it. Choice (A) is

incorrect because organisms that live on land still need to drink liquid water. Choice (B) is incorrect

because the polar ice caps contain frozen water that cannot support life as we know it. Choice (C) is

incorrect because all life as we know it requires liquid water to survive.

Page 76: Georgia Milestones Grade 6 EOG Assessment Guide

Georgia Milestones Grade 6 EOG Assessment Guide | Science

Georgia Department of Education November 2014 Page 75 of 101

All rights reserved.

Example Item 5

DOK Level: 3

Science Grade 6 Content Domain: Geology

Standard: S6E5. Students will investigate the scientific view of how the earth’s surface is formed. f.

Explain the effects of physical processes (plate tectonics, erosion, deposition, volcanic eruption, gravity)

on geological features including oceans (composition, currents, and tides).

The table shows information about some of the Hawaiian Islands. The ages of the islands to the

northwest are greater than the ages of the islands to the southeast.

Which reason BEST explains the difference in elevation of the Hawaiian Islands listed in the table?

A Laysan was once above water, but it eroded. B Oahu is located where the ocean floor is lower. C Volcanic eruptions caused Kure to move below sea level. D Hawaii's composition allows the island to float better than the other islands.

Correct Answer: A

Explanation of Correct Answer: The correct answer is choice (A) Laysan was once above water, but it

eroded. The older islands such as Laysan have been exposed longer to agents of erosion such as wind

and water. Over time, these islands likely became smaller until finally they disappeared below sea level.

Choice (B) is incorrect because where the ocean floor is lower, islands must be taller to reach above sea

level. Choice (C) is incorrect because volcanic eruptions are more likely to produce new land as lava

cools and solidifies. Choice (D) is incorrect because islands do not float; rather, they are tall mountains

whose bases are located underwater.

Page 77: Georgia Milestones Grade 6 EOG Assessment Guide

Georgia Milestones Grade 6 EOG Assessment Guide | Science

Georgia Department of Education November 2014 Page 76 of 101

All rights reserved.

Science Additional Sample Items

This section has two parts. The first part is a set of 10 sample items for the Science portion of the EOG

assessment. The second part contains a table that shows for each item the standard assessed, the DOK

level, the correct answer (key), and a rationale/explanation about the key and distractors. The sample

items can be utilized as a mini-test to familiarize students with the item formats found on the

assessment. All example and sample items contained in this guide are the property of the Georgia

Department of Education.

Page 78: Georgia Milestones Grade 6 EOG Assessment Guide

Georgia Milestones Grade 6 EOG Assessment Guide | Science

Georgia Department of Education November 2014 Page 77 of 101

All rights reserved.

Item 1

A scientist found a rock. She observed that the rock has layers and contains fossils.

Which type of rock did the scientist MOST LIKELY find?

A an igneous rock B a sedimentary rock C an igneous rock or a metamorphic rock D a metamorphic rock or a sedimentary rock

Item 2

A student researches weather patterns in Seattle, which is known for its foggy and wet weather. The

student uses a weather website to make the table.

Based on the information in the table, what MOST LIKELY causes fog and rain in Seattle?

A cool ocean air moves over land into air that is warmer B warm ocean air moves over land into air that is cooler C cool land air moves over the ocean into air that is warmer D warm land air moves over the ocean into air that is cooler

Page 79: Georgia Milestones Grade 6 EOG Assessment Guide

Georgia Milestones Grade 6 EOG Assessment Guide | Science

Georgia Department of Education November 2014 Page 78 of 101

All rights reserved.

Item 3

The pie chart shows the results of a study conducted to find out which major human activities are

responsible for soil erosion.

Which of these would MOST help reduce soil erosion?

A stopping the gathering of firewood B stopping the creation of new industries C reducing the amount of water used by farmers for irrigation D reducing the number of trees cut down by logging companies

Item 4

A student is observing a model of the oceanic floor at a geological museum.

Where in the model will the student MOST LIKELY find the presence of tectonic activity?

A oceanic basin B continental slope C oceanic ridge and trench D continental rise and shelf

Page 80: Georgia Milestones Grade 6 EOG Assessment Guide

Georgia Milestones Grade 6 EOG Assessment Guide | Science

Georgia Department of Education November 2014 Page 79 of 101

All rights reserved.

Item 5

An actor claims on his website that the Milky Way is between Earth and Mars. A student refers to a

science textbook to determine if this is true.

Which description would the science textbook MOST LIKELY give for the Milky Way?

A The Milky Way is the closest galaxy to our galaxy. B The Milky Way is the universe in which our galaxy is located. C The Milky Way is the closest solar system to our Solar System. D The Milky Way is the galaxy in which our Solar System is located.

Item 6

A student finds this model on a website.

According to the model, what season is occurring in the Southern Hemisphere?

A fall B spring C summer D winter

Page 81: Georgia Milestones Grade 6 EOG Assessment Guide

Georgia Milestones Grade 6 EOG Assessment Guide | Science

Georgia Department of Education November 2014 Page 80 of 101

All rights reserved.

Item 7

A student is making a diagram of Earth's layers.

How should his diagram be labeled to show the difference between Earth’s core and Earth’s crust?

A The core should be labeled as denser and hotter than the crust. B The core should be labeled as denser and colder than the crust. C The core should be labeled as denser than the crust but with the same temperature as the crust. D The core should be labeled as the same density as the crust but with a hotter temperature than the

crust.

Item 8

A student suggested that fossil fuels should be considered renewable resources because they were

formed by natural processes that still take place today.

Which of these BEST describes the main problem with the student’s suggestion?

A The plants and animals whose bodies became fossil fuels are now extinct. B It takes millions of years for fossil fuels to form from dead and decaying matter. C Sedimentation rates are lower today than in the past, so very few fossils are preserved. D High levels of atmospheric oxygen cause organic material to decay rather than form fossils.

Item 9

A student observed that a pole fixed near a beach remained visible throughout the day, but became

submerged in water at night. His friend believed that the sand was loose at night, causing the pole to

sink into the sand. The student did not agree with his friend's explanation.

Which of these MOST LIKELY caused the pole to be submerged at night?

A low tides from a lack of recent rainfall B high tides caused by the Moon’s gravity C smaller waves from a decrease in ocean currents D bigger waves from stronger winds blowing from the shore

Page 82: Georgia Milestones Grade 6 EOG Assessment Guide

Georgia Milestones Grade 6 EOG Assessment Guide | Science

Georgia Department of Education November 2014 Page 81 of 101

All rights reserved.

Item 10

A student is using a scientific website to do research on winds blowing in tropical regions for a report.

She discovers that warm, steady breezes are continually blowing in these regions.

Which of these MOST LIKELY causes these winds?

A The tropics receive more direct Sunrays, causing greater amounts of heating by the Sun. B The tropics receive more direct Sunrays, leading to the tropics always having the same temperature. C The tropics only receive direct Sunrays during spring and summer, causing warmer winds to form during fall and winter. D The tropics only receive direct Sunrays during spring and summer, leading to warm winds blowing in from the equator during fall and winter.

Page 83: Georgia Milestones Grade 6 EOG Assessment Guide

Georgia Milestones Grade 6 EOG Assessment Guide | Science

Georgia Department of Education November 2014 Page 82 of 101

All rights reserved.

Science Additional Sample Item Keys

Item Standard/ Element

Characteristics of Science

DOK Level

Correct Answer

Explanation

1 S6E5c S6CS9a 2 B

The correct answer is choice (B) a sedimentary rock. A sedimentary rock is made up of various sediments and other materials that have been deposited on Earth's surface and compacted or chemically cemented together. Sedimentary rocks often form in layers, and fossils may form from the remains of organisms trapped in these layers. Choices (A) and (C) are incorrect because igneous rocks form as molten rock cools and solidifies. Igneous rocks do not typically form in layers or contain fossils. Choice (D) is incorrect because metamorphic rocks form as heat and pressure cause chemical changes to rocks deep underground. Metamorphic rocks can show layers but typically do not contain fossils.

2 S6E4c S6CS6c 2 B

The correct answer is choice (B) warm ocean air moves over land into air that is cooler. In the student's table, the weather was foggy whenever the ocean temperature was significantly higher than the land temperature. On the only day when the land temperature was higher than the ocean temperature, the weather was not foggy. Choice (A) is incorrect because the table shows the opposite pattern: foggy weather happens on days when the ocean is warmer than the land. Choices (C) and (D) are incorrect because they describe changes to air over the ocean, not over Seattle.

3 S6E5i S6CS6c 2 D

The correct answer is choice (D) reducing the number of trees cut down by logging companies. According to the pie chart, overgrazing is most responsible for soil erosion, followed by deforestation. Because none of the answer choices addresses the problem of overgrazing, the best answer choice addresses the problem of deforestation. Choices (A), (B), and (C) are incorrect because compared to deforestation, gathering firewood, creating new industries, and using water for irrigation are less responsible for soil erosion.

Page 84: Georgia Milestones Grade 6 EOG Assessment Guide

Georgia Milestones Grade 6 EOG Assessment Guide | Science

Georgia Department of Education November 2014 Page 83 of 101

All rights reserved.

Item Standard/ Element

Characteristics of Science

DOK Level

Correct Answer

Explanation

4 S6E5f S6CS5a 2 C

The correct answer is choice (C) oceanic ridge and trench. Oceanic ridges and trenches are landforms that form along plate boundaries, where most tectonic activity happens. Choice (A) is incorrect because oceanic basins are broad, flat plains that typically form in the middle of tectonic plates. Choices (B) and (D) are incorrect because continental slopes, rises, and shelves are regions where the ocean floor slopes up toward dry land. These regions do not typically form along plate boundaries.

5 S6E1b S6CS7a 2 D

The correct answer is choice (D) The Milky Way is the galaxy in which our Solar System is located. The Milky Way contains billions of stars, including Earth's Sun, which is the center of our Solar System. Choices (B) and (C) are incorrect because the Milky Way is a galaxy, not a universe or solar system. Choice (A) is incorrect because the Milky Way is the galaxy in which we live.

6 S6E2c S6CS5b 2 C

The correct answer is choice (C) summer. In the diagram, the Southern Hemisphere is tilted toward the Sun. Therefore, it is receiving lots of sunlight and experiencing summer. Choice (D) is incorrect because the Southern Hemisphere experiences winter when it is tilted away from the Sun. Choices (A) and (B) are incorrect because fall and spring happen between these locations along Earth's orbit.

7 S6E5a S6CS5a 2 A

The correct answer is choice (A) The core should be labeled as denser and hotter than the crust. At Earth's core, the pressure from the planet's other layers is so great that the core remains solid metal despite its extremely high temperatures. Choices (B) and (C) are incorrect because Earth's core is the planet's hottest layer. Choice (D) is incorrect because Earth's core is the planet's densest layer.

Page 85: Georgia Milestones Grade 6 EOG Assessment Guide

Georgia Milestones Grade 6 EOG Assessment Guide | Science

Georgia Department of Education November 2014 Page 84 of 101

All rights reserved.

Item Standard/ Element

Characteristics of Science

DOK Level

Correct Answer

Explanation

8 S6E6b S6CS7a 3 B

The correct answer is choice (B) It takes millions of years for fossil fuels to form from dead and decaying matter. Choice (A) is incorrect because fossil fuels could still be generated from the remains of animals living today. Choice (C) is incorrect because fossil formation takes place over a very long period of time and sedimentation rates would not affect development of fossil fuels. Choice (D) is incorrect because the animal and plant remains that would become fossil fuels are those that would be buried rather than exposed to oxygen in the atmosphere.

9 S6E3d S6CS3d 2 B

The correct answer is choice (B) high tides caused by the Moon's gravity. As Earth rotates and the Moon orbits Earth, the Moon's gravity pulls on water at different places on Earth's surface. The water on the sides of Earth facing toward and away from the Moon extends farther from the planet than the water on the other sides of Earth. This effect creates high and low tides, respectively. Choices (C) and (D) are incorrect because wave size is not a factor that affects Earth's tides. Choice (A) is incorrect because, in this scenario, low tide happened during the day; at night, the water level rose high enough to submerge the pole.

10 S6E4b S6CS5a 3 A

The correct answer is choice (A) The tropics receive more direct Sunrays, causing greater amounts of heating by the Sun. Because Earth is round and tilts as it orbits the Sun, the tropics receive more direct sunlight than do regions closer to the poles. As the heated air rises and moves away from the tropics, it gradually cools and sinks back toward the ground and flows back to the tropics. The resulting convection cycles produce steady, warm winds in the tropics. Choice (B) is incorrect because temperatures change in the tropics, and everywhere else on Earth, as different parts of the planet face toward and away from the Sun. Choices (C) and (D) are incorrect because the tropics receive more direct Sunrays than the rest of Earth all year round, rather than in spring and summer.

Page 86: Georgia Milestones Grade 6 EOG Assessment Guide

Georgia Milestones Grade 6 EOG Assessment Guide | Social Studies

Georgia Department of Education November 2014 Page 85 of 101

All rights reserved.

SOCIAL STUDIES

Description of Test Format and Organization

The Georgia Milestones EOG assessment is primarily a criterion-referenced test, designed to provide

information about how well a student has mastered the grade-level state-adopted content standards in

Social Studies. Each student will receive one of four proficiency levels, depending on how well the

student has mastered the content standards. In addition to criterion-referenced information, the

Georgia Milestones measures will also include a limited sample of nationally norm-referenced items to

provide a signal of how Georgia students are achieving relative to their peers nationally. The norm-

referenced information provided is supplementary to the criterion-referenced proficiency designation

and will not be utilized in any manner other than to serve as a barometer of national comparison. Only

the criterion-referenced scores and proficiency designations will be utilized in the accountability metrics

associated with the assessment program (such as student growth measures, educator effectiveness

measures, or the CCRPI).

The Grade 6 Social Studies EOG assessment consists of a total of 75 selected-response items, 65 of

which are operational items (and contribute to a student’s criterion-referenced and/or norm-referenced

score) and 10 of which are field test items (newly written items that are being tried out and do not

contribute to the student’s score). The criterion-referenced score, and proficiency designation, is

comprised of 55 items, for a total of 55 points. Of the 65 operational items, 20 will be norm-referenced

and will provide a national comparison in the form of a national percentile rank. Ten of the items have

been verified as aligned to the course content standards by Georgia educators and will therefore

contribute to the criterion-referenced proficiency designation. The other 10 items will contribute only to

the national percentile rank and be provided as supplemental information. Only items that are aligned

to the state-adopted content standards will be utilized to inform the criterion-referenced score.

With the inclusion of the norm-referenced items, students may encounter items for which they have not

received direct instruction. These items will not contribute to the student’s criterion-referenced

proficiency designation; only items that align to the course content standards will contribute to the

criterion-referenced score. Students should be instructed to try their best should they ask about an item

that is not aligned to the content they have learned as part of the course.

Page 87: Georgia Milestones Grade 6 EOG Assessment Guide

Georgia Milestones Grade 6 EOG Assessment Guide | Social Studies

Georgia Department of Education November 2014 Page 86 of 101

All rights reserved.

Grade 6 Social Studies EOG Assessment Design

Description Number of

Items Points for CR1 Score

Points for NRT2 Feedback

CR Selected-Response Items 45 45 0

NRT Selected-Response Items 203 104 20

CR Field Test Items 10 0 0

Total Items/Points5 75 55 20 1CR—Criterion-Referenced: items aligned to state-adopted content standards 2NRT—Norm-Referenced Test: items that will yield a national comparison; may or may not be aligned to state-adopted content standards 3Of these items, 10 will contribute to both the CR scores and NRT feedback. The other 10 of these items will contribute to NRT feedback only and will not impact the student’s proficiency designation, scale score, or grade conversion. 4Alignment of national NRT items to course content standards was verified by a committee of Georgia educators. Only approved, aligned NRT items will contribute to a student’s CR proficiency designation, scale score, and grade conversion score. 5Total number of items contributing to CR score: 55; total points: 55; total number of items contributing to NRT feedback: 20; total points: 20

The test will be given in two sections. Students may have up to 70 minutes per section to complete

Sections 1 and 2. The total estimated testing time for the Grade 6 Social Studies EOG assessment ranges

from approximately 100 to 140 minutes. Total testing time describes the amount of time students have

to complete the assessment. It does not take into account the time required for the test examiner to

complete pre-administration and post-administration activities (such as reading the standardized

directions to students). Sections 1 and 2 must be scheduled to be administered on the same day in one

test session following the district’s testing protocols for the EOG measures (in keeping with state

guidance).

Content Measured

The Grade 6 Social Studies assessment will measure the standards that are enumerated for Grade 6 as

described on www.georgiastandards.org.

The content of the assessment is organized into four groupings, or domains, of standards for the

purposes of providing feedback on student performance. A content domain is a reporting category that

broadly describes and defines the content of the course, as measured by the EOG assessment. The

standards for Grade 6 Social Studies are grouped into four domains: Geography, Government and Civics,

Economics, and History. Each domain was created by organizing standards that share similar content

characteristics. The content standards describe the level of expertise that Grade 6 Social Studies

educators should strive to develop in their students. Educators should refer to the content standards for

a full understanding of the knowledge, concepts, and skills subject to be assessed on the EOG

assessment.

The approximate proportional number of points associated with each domain is shown in the following

table. A range of cognitive levels will be represented on the Grade 6 Social Studies EOG assessment.

Educators should always use the content standards when planning instruction.

Page 88: Georgia Milestones Grade 6 EOG Assessment Guide

Georgia Milestones Grade 6 EOG Assessment Guide | Social Studies

Georgia Department of Education November 2014 Page 87 of 101

All rights reserved.

Grade 6 Social Studies: Domain Structures and Content Weights

Domain Standard Approximate Weight

Geography

SS6G1 (1a, 1b) SS6G2

(2a) SS6G3

(3a, 3b) SS6G4

(4a, 4b, 4c) SS6G5

(5a) SS6G6

(6a, 6b) SS6G7

(7a)

SS6G8 (8a, 8b) SS6G9

(9a) SS6G10

(10a, 10b) SS6G11

(11a, 11b, 11c) SS6G12

(12a) SS6G13

(13a, 13b) SS6G14

(14a, 14b)

31%

Government and Civics

SS6CG1 (1a, 1b, 1c)

SS6CG2 (2a)

SS6CG3 (3a)

SS6CG4 (4a, 4b, 4c)

SS6CG5 (5a, 5b) SS6CG6

(6a, 6b, 6c) SS6CG7

(7a)

15%

Economics

SS6E1 (1a, 1b, 2c)

SS6E2 (2a, 2b, 2c, 2d)

SS6E3 (3a, 3b, 3c, 3d)

SS6E4 SS6E5

(5a, 5b, 5c) SS6E6

(6a, 6b)

SS6E7 (7a, 7b, 7c, 7d)

SS6E8 (8a, 8b, 8c)

SS6E9 (9a, 9b, 9c)

SS6E10 (10a, 10b, 10c,

10d)

25%

History

SS6H1 (1a, 1b) SS6H2

(2a, 2b, 2c) SS6H3

(3a, 3b) SS6H4

(4a, 4b) SS6H5

(5a)

SS6H6 (6a, 6b, 6c, 6d)

SS6H7 (7a, 7b, 7c)

SS6H8 (8a)

SS6H9 (9a, 9b)

29%

Page 89: Georgia Milestones Grade 6 EOG Assessment Guide

Georgia Milestones Grade 6 EOG Assessment Guide | Social Studies

Georgia Department of Education November 2014 Page 88 of 101

All rights reserved.

Item Types

The Social Studies portion of the Grade 6 EOG assessment consists of selected-response items only.

A selected-response item, sometimes called a multiple-choice item, is defined as a question, problem, or

statement that appears on a test followed by several answer choices, sometimes called options or

response choices. The incorrect choices, called distractors, usually reflect common errors. The student’s

task is to choose, from the alternatives provided, the best answer to the question posed in the stem (the

question). The Social Studies selected-response items will have four answer choices.

Social Studies Example Items

Example items, which are representative of three DOK levels across various Grade 6 Social Studies

content domains, are provided on the following pages. All example and sample items contained in this

guide are the property of the Georgia Department of Education.

Page 90: Georgia Milestones Grade 6 EOG Assessment Guide

Georgia Milestones Grade 6 EOG Assessment Guide | Social Studies

Georgia Department of Education November 2014 Page 89 of 101

All rights reserved.

Example Item 1

DOK Level: 1

Social Studies Grade 6 Content Domain: Geography

Standard: SS6G1. The student will locate selected features of Latin America and the Caribbean. a. Locate

on a world and regional political-physical map: Amazon River, Caribbean Sea, Gulf of Mexico, Pacific

Ocean, Panama Canal, Andes Mountains, Sierra Madre Mountains, and Atacama Desert.

Look at the map.

Which letter on the map shows the location of the Panama Canal?

A A B B C C D D

Correct Answer: C

Explanation of Correct Answer: The correct answer is choice (C) C. The Panama Canal connects the

Atlantic and Pacific Oceans through the Isthmus of Panama. Choice (A) is incorrect because it shows

Mexico. Choice (B) is incorrect because it shows Guatemala. Choice (D) is incorrect because it shows

Bolivia.

Page 91: Georgia Milestones Grade 6 EOG Assessment Guide

Georgia Milestones Grade 6 EOG Assessment Guide | Social Studies

Georgia Department of Education November 2014 Page 90 of 101

All rights reserved.

Example Item 2

DOK Level: 2

Social Studies Grade 6 Content Domain: History

Standard: SS6H6. The student will analyze the impact of European exploration and colonization on

various world regions. d. Explain the impact of European empire building in Africa and Asia on the

outbreak of WWI.

Which of these contributed to the outbreak of World War I?

A the drought and plague that spread around the world B the spread of communism in Russia and other regions C the formation of the League of Nations by European and other nations D the growing competition among European powers to colonize Africa and Asia

Correct Answer: D

Explanation of Correct Answer: The correct answer is choice (D) the growing competition among

European powers to colonize Africa and Asia. Several European countries wanted to take control of

lands in resource-rich Africa and Asia, leading to global conflicts. Choices (A), (B), and (C) are incorrect

because these were not factors in the outbreak of World War I.

Page 92: Georgia Milestones Grade 6 EOG Assessment Guide

Georgia Milestones Grade 6 EOG Assessment Guide | Social Studies

Georgia Department of Education November 2014 Page 91 of 101

All rights reserved.

Example Item 3

DOK Level: 2

Social Studies Grade 6 Content Domain: Government and Civics

Standard: SS6CG2. The student will explain the structures of national governments in Latin America and

the Caribbean. a. Compare the federal-republican systems of the Federative Republic of Brazil (Brazil)

and the United Mexican States (Mexico) to the dictatorship of the Republic of Cuba (Cuba),

distinguishing the form of leadership and the role of the citizen in terms of voting and personal

freedoms.

How are the governments of Brazil and Mexico similar?

A A single individual makes all government decisions. B A small group of citizens runs the entire government. C Citizens vote for government officials in a national election. D Government officials inherit their positions through the royal family.

Correct Answer: C

Explanation of Correct Answer: The correct answer is choice (C) Citizens vote for government officials in

a national election. Both Brazil and Mexico are led by a president who is elected by popular vote.

Choices (A), (B), and (D) are incorrect because these features describe neither Brazilian nor Mexican

government.

Page 93: Georgia Milestones Grade 6 EOG Assessment Guide

Georgia Milestones Grade 6 EOG Assessment Guide | Social Studies

Georgia Department of Education November 2014 Page 92 of 101

All rights reserved.

Example Item 4

DOK Level: 3

Social Studies Grade 6 Content Domain: Economics

Standard: SS6E3. The student will describe factors that influence economic growth and examine their

presence or absence in Latin America. c. Describe the role of natural resources in a country’s economy.

Look at the chart.

Facts about Venezuela

Exports Petroleum, bauxite, aluminum, steel, chemicals, agricultural products, and basic manufactured products

Imports Raw materials, machinery, transport equipment, and construction materials

Gross Domestic Product (GDP) per Capita $13,200

Literacy Rate 93%

Unemployment Rate 10.9%

Which of these would MOST LIKELY happen if Venezuela used up all its mineral resources?

A Its labor force would increase. B Its manufacturing would increase. C Its unemployment rate would decrease. D Its gross domestic product (GDP) would decrease.

Correct Answer: D

Explanation of Correct Answer: The correct answer is choice (D) Its gross domestic product (GDP) would

decrease. GDP is a calculation of the value of goods and services produced in a country, so a country

that uses up all its mineral resources would likely produce fewer goods and see its GDP fall. Choices (A),

(B), and (C) are incorrect because a decrease in mineral resources would likely have the opposite effect

of those described.

Page 94: Georgia Milestones Grade 6 EOG Assessment Guide

Georgia Milestones Grade 6 EOG Assessment Guide | Social Studies

Georgia Department of Education November 2014 Page 93 of 101

All rights reserved.

Example Item 5

DOK Level: 3

Social Studies Grade 6 Content Domain: Economics

Standard: SS6E3. The student will describe factors that influence economic growth and examine their

presence or absence in Latin America. a. Explain the relationship between investment in human capital

(education and training) and gross domestic product (GDP).

Look at the chart.

Based on the information in the chart, what is the BEST way for a country to grow its economy?

A increase its population B protect its environment C invest in human capital D colonize neighboring countries

Correct Answer: C

Explanation of Correct Answer: The correct answer is choice (C) invest in human capital. The chart

shows that the higher a country's literacy rate is, the higher its GDP per capita, which suggests that

having an educated population better helps a country grow its economy. Choice (A) is incorrect because

Haiti has the highest population in the chart but the lowest GDP per capita. Choices (B) and (D) are

incorrect because the chart does not provide information about the environment or neighboring

countries.

Page 95: Georgia Milestones Grade 6 EOG Assessment Guide

Georgia Milestones Grade 6 EOG Assessment Guide | Social Studies

Georgia Department of Education November 2014 Page 94 of 101

All rights reserved.

Social Studies Additional Sample Items

This section has two parts. The first part is a set of 10 sample items for the Social Studies portion of the

EOG assessment. The second part contains a table that shows for each item the standard assessed, the

DOK level, the correct answer (key), and a rationale/explanation about the key and distractors. The

sample items can be utilized as a mini-test to familiarize students with the item formats found on the

assessment. All example and sample items contained in this guide are the property of the Georgia

Department of Education.

Page 96: Georgia Milestones Grade 6 EOG Assessment Guide

Georgia Milestones Grade 6 EOG Assessment Guide | Social Studies

Georgia Department of Education November 2014 Page 95 of 101

All rights reserved.

Item 1

Look at the map.

Which of these countries is identified by the star on the map?

A Italy B Spain C Finland D Switzerland

Item 2

Which of these describes an entrepreneur?

A a soybean farmer in Mexico B a cook buying beef in Argentina C a tourist visiting the rain forest in Brazil D a bank loaning money to a student in Canada

Page 97: Georgia Milestones Grade 6 EOG Assessment Guide

Georgia Milestones Grade 6 EOG Assessment Guide | Social Studies

Georgia Department of Education November 2014 Page 96 of 101

All rights reserved.

Item 3

Australia has a parliamentary democracy. Which of these is the head of government in Australia?

A dictator B president C prime minister D military general

Item 4

Look at the headline.

Which positive economic impact would these discoveries have on the region mentioned in the

headline?

A an increase in trade B fewer environmental concerns C an increase in farm production D fewer people moving to the area

Page 98: Georgia Milestones Grade 6 EOG Assessment Guide

Georgia Milestones Grade 6 EOG Assessment Guide | Social Studies

Georgia Department of Education November 2014 Page 97 of 101

All rights reserved.

Item 5

Look at the chart.

Which of these factors plays the GREATEST role in determining where Australia is placed on the chart?

A the amount it trades with other nations B the level of unemployment in the country C the type of goods produced by the country D the percentage of industries that are privately owned

Item 6

Which country had the MOST influence on the primary language spoken in Latin America today?

A Italy B Spain C France D Germany

Item 7

Which of these describes Prince Henry the Navigator?

A He spread Christianity to Asia. B He established a colony in South America. C He started the Industrial Revolution in Europe. D He sponsored many sea voyages along the coast of Africa.

Page 99: Georgia Milestones Grade 6 EOG Assessment Guide

Georgia Milestones Grade 6 EOG Assessment Guide | Social Studies

Georgia Department of Education November 2014 Page 98 of 101

All rights reserved.

Item 8

Look at the circle graph.

What conclusion can be supported by the information in the circle graph?

A African explorers spread their religion to a large portion of the Australian population. B English colonization played a major role in the development of religion in Australia. C Chinese traders introduced their religion to coastal cities and it spread across much of Australia. D Aboriginal natives of Australia maintained their religions and were able to convert colonists over

time.

Item 9

While doing research for a report, Jeff found this information:

This system was created to improve political stability.

This system was created to promote the ability of countries to work together.

This system made trade easier among member nations.

Which would be the BEST title for Jeff's report?

A How Nazism Gained Influence B The Iron Curtain and Its Benefits C The Creation of the Treaty of Versailles D Why the European Union Was Established

Page 100: Georgia Milestones Grade 6 EOG Assessment Guide

Georgia Milestones Grade 6 EOG Assessment Guide | Social Studies

Georgia Department of Education November 2014 Page 99 of 101

All rights reserved.

Item 10

Read the information in the box.

helped plan a revolution against Spain

rang church bells to call people to revolt

known as the father of Mexico’s independence

Which person is described in the box?

A Simon Bolivar B Hernando Cortes C Miguel Hidalgo D Toussaint L’Ouverture

Page 101: Georgia Milestones Grade 6 EOG Assessment Guide

Georgia Milestones Grade 6 EOG Assessment Guide | Social Studies

Georgia Department of Education November 2014 Page 100 of 101

All rights reserved.

Social Studies Additional Sample Item Keys

Item Standard/ Element

DOK Level Correct Answer

Explanation

1 SS6G8b 1 B

The correct answer is choice (B) Spain. Spain is located in southwestern Europe on the Iberian Peninsula. Choices (A) and (D) are incorrect because Italy and Switzerland are located further east. Choice (C) is incorrect because Finland is located further north and east.

2 SS6E3d 2 A

The correct answer is choice (A) a soybean farmer in Mexico. An entrepreneur is a person who operates a business that produces goods or services. A farmer is an entrepreneur. Choice (B) is incorrect because this is an example of a customer buying a product. Choice (C) is incorrect because a tourist is not operating a business. Choice (D) is incorrect because an entrepreneur is a person, not a business.

3 SS6CG7a 1 C

The correct answer is choice (C) prime minister. Australia's government is led by a prime minister. Choices (A), (B), and (D) are incorrect because Australia's government is not led by a dictator, president, or military general.

4 SS6G6b 2 A

The correct answer is choice (A) an increase in trade. The discovery of a valuable natural resource such as oil would increase Canada's ability to trade with other countries. Choice (B) is incorrect because the discovery of oil would more likely increase environmental concerns. Choice (C) is incorrect because the discovery of oil would not likely have a large effect on farm production. Choice (D) is incorrect because the discovery of oil would likely lead to more people moving to the area.

5 SS6E8c 3 D

The correct answer is choice (D) the percentage of industries that are privately owned. In a command economy, the factors of production are owned by the government. In a market economy, the factors of production are privately owned, with minimal government involvement. In Australia, a larger percentage of industries are privately owned than government owned, so it is closer to a pure market economy than pure command. Choices (A), (B), and (C) are incorrect because these factors do not affect whether Australia is a command or market economy.

Page 102: Georgia Milestones Grade 6 EOG Assessment Guide

Georgia Milestones Grade 6 EOG Assessment Guide | Social Studies

Georgia Department of Education November 2014 Page 101 of 101

All rights reserved.

Item Standard/ Element

DOK Level Correct Answer

Explanation

6 SS6H2b 1 B

The correct answer is choice (B) Spain. Many Latin American countries were originally colonized by Spain, so the primary language spoken in these regions is Spanish. Choices (A), (C), and (D) are incorrect because French, German, and Italian are not the primary languages spoken in Latin America today.

7 SS6H6a 2 D

The correct answer is choice (D) He sponsored many sea voyages along the coast of Africa. Prince Henry of Portugal funded several expeditions to Africa during the early 1400s that led to important discoveries of new lands and natural resources. Choices (A), (B), and (C) are incorrect because they do not describe accomplishments made by Prince Henry the Navigator.

8 SS6G14a 3 B

The correct answer is choice (B) English colonization played a major role in the development of religion in Australia. English colonists were primarily Christian, and they spread their religion when they populated Australia, explaining why Christianity is the dominant religion in the country. Choices (A), (C), and (D) are incorrect because the graph does not reflect the religions held by African explorers, Chinese traders, or Aboriginal natives of Australia.

9 SS6CG5b 2 D

The correct answer is choice (D) Why the European Union Was Established. The European Union was created in 1993 to improve political stability, promote cooperation, and make trade easier between European countries. Choices (A), (B), and (C) are incorrect because the information in the box does not describe Nazism, the Iron Curtain, or the Treaty of Versailles.

10 SS6H2c 2 C

The correct answer is choice (C) Miguel Hidalgo. Hidalgo was an important figure in Mexican history who was known as the father of Mexico's independence because he helped plan and execute a revolution against Spain. Choices (A), (B), and (D) are incorrect because the items in the list do not describe these historical figures.